Download as docx, pdf, or txt
Download as docx, pdf, or txt
You are on page 1of 102

Infectious Disease:

HIV:

Acute Viral Infection:

Although many patients are asymptomatic, dramatic early elevations in viral load often result in transient
manifestations 2-4 weeks after inoculation.

Symptoms can closely resemble infectious mononucleosis and commonly include fever, nontender
lymphadenopathy, sore throat, headache, fatigue, and myalgia.  Two distinctive features are a painful
mucocutaneous ulcer (shallow, discrete, white base) and a generalized maculopapular rash that may include
the palms and soles.  Laboratory testing often shows leukopenia and thrombocytopenia.

DX: Diagnosis is made with a combination of fourth-generation HIV serologic testing (HIV antigen plus HIV-
1/HIV-2 antibody) and HIV viral load. 

Patients at high risk of HIV exposure should receive a screening test that combines detection of HIV antigen
(p24) and HIV-1/HIV-2 antibody.  False-negative results may occur during the first 4 weeks of infection due
to low titers of antigen and antibody (window period).

RX: Prior to the initiation of antiretroviral therapy, testing for coinfection with hepatitis B virus is required as
some antiretroviral medications have dual activity against both hepatitis B virus and HIV.  These medications
are often used preferentially in patients with both infections. Patients are also typically screened for
tuberculosis, hepatitis C virus, and sexually transmitted pathogens (eg, Treponema pallidum, Neisseria
gonorrhoeae).

Patient with a new diagnosis of HIV should now have genotypic drug-resistance testing followed by the
initiation of combination antiretroviral therapy (ART). ART is recommended for all patients with HIV
regardless of CD4 count.  Early initiation of ART results in the following:

-Improved probability of restoring normal CD4 counts and normal CD4/CD8 ratio
-Reduction in immune activation and system-wide inflammation
-Reduction in AIDS-defining (eg, opportunistic infections) and non-AIDS-defining (eg, cardiovascular disease,
kidney disease, malignancy) complications

1
-Improved quality of life

Patients on ART also have lower viral concentrations in genital secretions, reducing the risk of transmission to
sexual partners by up to 96%.

Virologic failure is defined as a failure to achieve a viral load (VL) <200 copies/mL within 24 weeks (6
months) of antiretroviral therapy (ART) and may be due to drug resistance or noncompliance. 

In general, VL in treatment-naïve patients initiated on ART is expected to decrease to the following:


-<5,000 copies/mL by 4 weeks (logarithmic drop)
-<500 copies/mL by 8-16 weeks
-<50 copies/mL by 16-24 weeks .

D/D:
-Herpes simplex virus can cause fever, fatigue, and headache; however, patients typically have multiple
ulcerating lesions and tender lymphadenopathy (unlike this patient).

- Infectious mononucleosis (Epstein-Barr virus, cytomegalovirus) can cause fever, lymphadenopathy, fatigue,
and sore throat.  Mucocutaneous ulceration and a maculopapular rash that includes the palms and soles are
quite uncommon.

-Influenza often manifests with fever, fatigue, myalgias, sore throat, and headache.  However, influenza is a
respiratory infection typically associated with respiratory tract symptoms such as cough and nasal congestion.

- Secondary syphilis can cause fever, headache, nontender lymphadenopathy, fatigue, and a maculopapular
rash that includes the palms and soles.  A single, painful mucocutaneous lesion would be atypical (syphilitic
chancres are nontender and a marker of primary syphilis).

IRIS:
Patients with HIV may develop a transient worsening of infectious symptoms several weeks after the initiation
of antiretroviral therapy due to immune reconstitution inflammatory syndrome (IRIS).  IRIS arises due to the
potent immune recovery that quickly occurs after the initiation of antiretroviral therapy.  IRIS is typically self-
limited and requires no alteration to ongoing treatment.  Anti-inflammatory medications are sometimes added
if IRIS symptoms are bothersome.

IRIS is particularly common when patients are started on treatment for both Mycobacterium tuberculosis and
HIV. Only patients with life-threatening symptoms of IRIS (eg, an enlarging mass threatening the airway)
require immediate antiretroviral discontinuation.

Other Opportunistic infections in HIV:

Cytomegalovirus is a common opportunistic infection in those with advanced HIV (CD4 count <100/mm3)
and may cause viremia or tissue-invasive disease (eg, retinitis, colitis).  Cytomegalovirus testing is only
required when an active infection is suspected

TB: Risk of tuberculosis reactivation is much greater in patients with HIV.  All newly diagnosed patients
should receive latent tuberculosis screening with skin testing or interferon-gamma release assay.  Sputum
samples are only required if screening is positive and the patient is symptomatic (eg, night sweats, weight loss,
cough) or chest x-ray is abnormal.

PCP: Trimethoprim-sulfamethoxazole (TMP-SMX) is used as primary prophylaxis to prevent Pneumocystis


pneumonia in patients with HIV with CD4 counts <200/mm3 and to prevent toxoplasmosis in those with
positive serology and CD4 counts <100/mm3. 

2
HIV In Health care personnel (HCP): HCPP are at risk for occupational acquisition of HIV infection if
body fluids from a patient with HIV penetrate the cutaneous barrier (percutaneous injury, nonintact skin) or
mucous membranes.  Although overall transmission risk is quite low, the likelihood of infection is increased
when the source patient has a high viral load or the exposure is large/deep.
All potential exposures should be evaluated to determine whether postexposure prophylaxis (PEP) with 3-drug
antiretroviral therapy is required.  Generally, PEP is offered to patients with any risk of occupational
transmission as guided by exposure type and body fluid involved.

HCP with the following occupational exposures are at risk for acquiring HIV:

-Percutaneous injury from a contaminated needlestick or sharp object (eg, scalpel)

-Mucous membrane (eg, eyes) or nonintact skin exposure to infected body fluids

However, these exposure types put HCP at risk only when the following fluids are involved:

High risk:  Blood, fluids contaminated with visible blood, semen, and vaginal secretions

Possible risk:  Cerebrospinal, synovial, pleural, pericardial, and amniotic fluids. Body fluids such as urine,
feces, tears, and vomitus are considered noninfectious if no visible blood is present.

Management:
PEP is most effective if initiated within 1-2 hours of exposure. Most PEP recommendations involve 3-drug
antiretroviral therapy. Monotherapy and more extended treatment are not advised. 

In case of cutaneous wounds should be thoroughly washed with soap and water, and exposed mucous
membranes require irrigation.  Laboratory testing along with initiation of postexposure prophylaxis with
combination antiretroviral therapy should begin 1-2 hours after exposure and continue for 4 weeks. 
Fourth-generation HIV testing should then be repeated at 6 weeks and 4 months to evaluate for
seroconversion.

HCP are also at risk of exposure to hepatitis B and hepatitis C, which are far more transmissible than HIV.

Non-occupational HIV post-exposure prophylaxis:


Postexposure HIV prophylaxis with highly active antiretroviral therapy for 28 days is recommended following
high-risk exposure to an HIV-infected individual.  Triple drug therapy is preferred and should be started within
72 hours of exposure. However, 2-drug regimens may be considered for individual cases when better
tolerability or compliance is needed.

3
HIV During pregnancy:

The risk of perinatal HIV transmission directly correlates with maternal HIV viral load.  Antiretroviral
therapy (typically, highly active antiretroviral therapy [HAART]) is continued throughout pregnancy as it is
safe, treats maternal disease, and prevents perinatal transmission by suppressing viral replication and
decreasing the maternal viral load.

4
Modification of an effective antiretroviral regimen is generally avoided due to the risk of emerging drug
resistance, loss of viral suppression, and during pregnancy, perinatal transmission However, changes are
indicated if significant side effects or inadequate viral suppression is present.  During pregnancy, alterations in
drug metabolism and distribution volume often necessitate changes in dosing or medication to maintain a low
viral load.  Therefore, viral loads and CD4 counts are evaluated every 3 months.

Mode of delivery: Adequate virologic control is important during pregnancy as the delivery modality (eg,
cesarean, vaginal) is based on the third-trimester viral load.  In women with a viral load of <1,000 copies/mL,
the risk of perinatal transmission is low irrespective of delivery modality, so a vaginal delivery is preferred.  In
contrast, when the viral load is >1,000 copies/mL, cesarean delivery is indicated as there is a higher risk for
perinatal transmission with vaginal delivery.

Intrapartum: antiretroviral therapy is indicated to further decrease the risk of perinatal HIV transmission. 
Patients with a viral load of <1,000 copies/mL continue HAART.  In contrast, patients with a viral load of
>1,000 copies/mL receive intrapartum intravenous zidovudine in addition to the HAART regimen.

Breast feeding in HIV: HIV-positive women in resource-rich countries are advised to feed their infants with
formula rather than breast milk, regardless of maternal or infant antiretroviral therapy.  In contrast, women in
resource-poor countries are recommended to breastfeed due to high rates of infant morbidity and mortality
from water-borne infectious diseases associated with formula feeding.  
In addition, most antiretroviral drugs are excreted in breast milk in significant enough quantities to provide
passive infant protection and decrease the risk of postnatal HIV transmission. In all countries, infants born to
HIV-positive mothers should receive zidovudine prophylaxis for >6 weeks while the mother continues highly
active antiretroviral therapy for indefinite viral suppression.

HIV-positive children should attend school in a normal manner (ie, commingling with other students in the
classroom and on the playground) and may play sports.  Any disclosure of HIV status by the family is
voluntary.

STD Screening:

Recommendations for sexually transmitted infection (STI) screening are fairly nuanced and depend on many
factors, including gender, sexual orientation, age, high-risk behavior, and symptoms.  However, most STI
screening guidelines recommend that patients age 13-65 receive HIV screening with a fourth-generation HIV
test (p24 antigen and HIV antibodies), regardless of the presence or absence of risk factors.  Early
identification of HIV allows for the initiation of antiretroviral therapy, administration of appropriate
vaccinations, and early suppression of viral load (which reduces transmission risk).

Asymptomatic men who have sex with women and do not engage in high-risk behavior (eg, encounters with
sex workers) generally do not require additional STI screening.  If a sexual partner has a known or previous
STI, then targeted testing is often performed.

HCV: As such, HCV is not a routine part of STI screening.  Testing for HCV is recommended in those with a
risk factor (eg, history of injection drug use, blood transfusion before 1992, unregulated tattoo), with elevated
alanine aminotransferase, or in high-prevalence groups (eg, HIV infection, hemodialysis, born 1945-1965,
incarcerated).

Clamydia: High-risk (eg, men who have sex with men) or symptomatic men should receive testing.  In
general, compared to women, men are more likely to be symptomatic and less likely to have long-term
sequelae from chlamydial infections.

HIV Lipodystrophy:

5
Patients with HIV-associated lipodystrophy develop alterations in fat deposition with no change in lean tissue
mass.  Manifestations include one or both of the following:

 Lipoatrophy – loss of subcutaneous fat from the face, arms, legs, abdomen, and/or buttocks, which
can give patients a skeleton-like appearance.  Lipoatrophy is particularly associated with nucleoside
reverse transcriptase inhibitors (eg, stavudine, zidovudine).
 Fat accumulation – dorsocervical fat accumulation leads to a "buffalo hump" and visceral
abdominal fat accumulation leads to increased abdominal girth (despite minimal subcutaneous
abdominal fat).  Fat accumulation is not clearly linked with a particular antiretroviral drug and can
occur with any regimen.

Patients with HIV-associated lipodystrophy have abnormal lipid and glucose metabolism, which often leads to
insulin resistance, dyslipidemia, and an increased risk of cardiovascular disease.

D/D:
-Cushing syndrome (adrenal hyperplasia)often manifests with wasting of the extremities and increased central
fat.  However, most patients also have other symptoms, including proximal muscle weakness, facial plethora,
easy bruising, and/or purple abdominal striae. 

Antiretroviral therapy (ART) can cause marked dyslipidemia including:

-Increased total cholesterol, LDL, and triglycerides


-Reduced high-density lipoprotein

The most commonly used statins in patients with HIV are rosuvastatin, atorvastatin, and pravastatin.  Choice of
specific statin is typically guided by current ART regimen to limit drug-drug interactions.

HIV-associated thrombocytopenia (HIV-TP):

HIV can be associated with deficiencies in all hematologic cell lines.  HIV-TP is rarely associated with
bleeding (<5% of patients have platelet counts <50,000/mm3).  ART generally improves the
thrombocytopenia.

Tuberculosis:

Latent TB (LTBI):

Health care providers with positive tuberculosis screening should receive chest x-ray and symptom review to
determine if they have latent or active disease.  Those with no symptoms or chest x-ray abnormalities (eg,
infiltrate, cavitation) are considered to have latent tuberculosis infection and should be offered treatment with
isoniazid daily for 9 months. LTBI is noninfectious, and individuals may continue their normal daily lives
without restrictions.  However, because HCP work in congregant settings, treatment (eg, isoniazid) should be
offered. As such, most organizations recommend 2-step pre-employment testing for LTBI; individuals with
negative initial skin tests should have repeat testing in 1-3 weeks. 

6
When there are no radiographic signs of active infection in this patient, the presence of active tuberculosis
cannot be entirely excluded radiographically and correlation with the patient's symptoms is needed.  Ancillary
tests such as sputum culture may be needed before determining that the patient has no active tuberculosis. 
Given the radiographic findings and absence of clinical symptoms in patients, latent tuberculosis is most
likely. 

RX: Treatment is still indicated for latent tuberculosis since the infection can become reactivated should the
patient become immunocompromised.  The regimen of choice for treatment of latent tuberculosis is isoniazid
for 9 months.

Rifampin for 4 months is an alternative treatment for latent tuberculosis, although the combination of rifampin
with isoniazid is not recommended.

Prior BCG vaccination should rarely cause more than 15 mm induration with PPD skin testing

In Children:

Children with latent tuberculosis infection require treatment to prevent conversion to active disease.  Daily
isoniazid for 9 months is the standard of care.  If isoniazid resistance is suspected, the best alternate therapy is
daily rifampin for 4-6 months.

7
Common Diagnostics tests for TB:

Suspicion of active TB should be raised when individuals with epidemiologic risk factors for exposure (eg,
substance abuse, homelessness, birth in a TB-endemic region) develop characteristic clinical manifestations
(eg, fever, cough >2 weeks, weight loss).
DX:
-CXR: Diagnostic evaluation begins with chest x-ray to determine if there are signs of active disease such as
upper lobe cavitation (70%-80%), hilar lymphadenopathy, or pleural effusion.  Patients with suspicious x-ray
findings are definitively diagnosed when Mycobacterium tuberculosis is isolated in body fluid or tissue (lung,
pleura).
-Sputum sampling provides the least invasive and costly route for microbial confirmation.  Three single
sputum samples (spontaneous or induced) are submitted in 8- to 24-hour intervals with at least 1 early-morning
sample.  Sputum should be sent for acid-fast bacillus smear, mycobacterial culture, and nucleic acid
amplification testing.

Although patients with suspected pulmonary TB should also undergo tuberculin skin testing or interferon-
gamma release assay, these tests cannot distinguish between active and latent disease and are ordered primarily
to support the diagnosis (positive test results suggest exposure)

-Bronchoscopy with bronchoalveolar lavage is more invasive and expensive than sputum sampling and is
generally reserved for patients who are unable to produce adequate expectorated or induced sputum, have
negative sputum studies with a high suspicion for active TB, or have a possible alternate diagnosis that
requires bronchoscopy for evaluation.
-Transthoracic needle aspiration is invasive and it is typically reserved for patients who remain undiagnosed
after these tests.

8
Sputum acid-fast bacillus (AFB) smear is the most rapid, least expensive, and widely used diagnostic test for
pulmonary tuberculosis (TB).  However, this test cannot be used to definitively rule in/out the diagnosis due to:

 Low sensitivity (45%-80%) - microscopic detection of AFB requires a high burden of organisms
(>10,000 bacilli/mL), so false negatives are common
 Poor differentiation - cannot distinguish between tuberculous and nontuberculous mycobacterium
(both are AFB positive), so confirmatory testing with mycobacterial culture or nucleic acid
amplification (NAA) testing is required

Diagnosis of TB can be definitively established or excluded with one or both of the following:

 Mycobacterial culture - takes 2-6 weeks but is considered the gold standard for microbial
confirmation and drug susceptibility
 NAA testing - highly sensitive (95%) and specific (98%), and results return rapidly (<48 hours);
unlike AFB smears, NAA testing can distinguish between M tuberculosis and nontuberculous
mycobacteria

Anergy to tuberculin antigen can be caused by immunosuppression (eg, HIV) or waning immunity and may
lead to false-negative (not false-positive) tuberculin skin testing.

Educational objective:
Sputum acid-fast bacilli smear is a common initial diagnostic test for active pulmonary tuberculosis.  Positive
testing supports the diagnosis and may lend credence to empiric treatment.  However, this test is limited by
low sensitivity and a lack of distinction between tuberculous and nontuberculous mycobacterium, so it cannot
be used to definitively rule in/out the diagnosis.  Additional testing with mycobacterial culture or nucleic acid
amplification is required for confirmation.

Individuals who work in the health care setting should receive pre-employment tuberculosis (TB) screening to
establish baseline TB status.  Two main screening tools are used: tuberculin skin test and interferon-gamma
release assay (IGRA).  An IGRA should be used to screen for TB in most patients with previous bacillus
Calmette-Guérin vaccine as these patients may have a false-positive tuberculin skin test.

IGRA: Although roughly comparable in accuracy, IGRA has 3 main advantages over TST:

 No false positives in patients with previous bacillus Calmette-Guérin (BCG) vaccine:  


 No need for a return visit to measure induration size
 IGRA is not subject to the "booster" phenomenon.

This patient who previously received a BCG vaccine should be screened with an IGRA, not a
TST.

Transmission of TB:
Patients with active pulmonary tuberculosis (TB) may transmit the bacterium to close contacts via aerosolized
droplets for up to 3 months prior to the onset of symptoms. Patients with active pulmonary tuberculosis are at
risk of transmitting Mycobacterium tuberculosis to close contacts for up to 3 months prior to the onset of
symptoms.  Health care personnel are also at risk until appropriate infection control measures (airborne
isolation, N95 mask) are undertaken.  Those with potential exposure should receive screening with tuberculin
skin testing or interferon-gamma release assay.  If negative, screening should be repeated at 8-10 weeks.

Tuberculous Meningitis:

9
Manifestations typically begin with 2-3 weeks of prodromal symptoms (eg, headache, low-grade fever,
lassitude) followed by progressive signs of meningeal irritation (eg, nuchal rigidity, vomiting, confusion). 
Cranial nerve palsies, coma, and seizures may occur.  Choroidal tubercles (yellow-white nodules near the
optic disc) are often detected via funduscopic examination.

Suspicion for tuberculous meningitis is typically raised when brain imaging reveals basilar meningeal
enhancement and cerebrospinal fluid (CSF) examination shows the following:

 Elevated protein (>250 mg/dL, generally 100-500 mg/dL)


 Low glucose (<10 mg/dL)
 Lymphocytic pleocytosis
 Elevated adenosine deaminase

DX: Definitive diagnosis requires serial lumbar punctures with CSF examination for acid-fast bacilli using
smear and culture.

RX: Prolonged treatment is required with 2 months of 4-drug therapy (isoniazid, rifampin, pyrazinamide,
and either a fluoroquinolone or injectable aminoglycoside), followed by 9-12 months of continuation therapy
(isoniazid plus rifampin).

As antitubercular treatment can result in a transient worsening of central nervous system inflammation,
patients with tuberculous meningitis are typically given 8 weeks of adjuvant glucocorticoid therapy
(dexamethasone or prednisone).  This significantly reduces morbidity and mortality.

D/D:
-Cytomegalovirus can cause a mononucleosis-type syndrome with fever, malaise, and lymphadenopathy. 
Retinitis and encephalitis may occur but are rare in patients with intact immunity. 

10
-Cryptococcal meningitis can be diagnosed with India ink preparation of CSF.  Although most cases occur in
patients with advanced HIV, immunocompetent individuals can develop subacute meningitis from
Cryptococcus in rare cases.  Basilar meningeal enhancement is uncommon.

-Toxoplasmosis infection is usually asymptomatic in immunocompetent individuals, but it may occasionally


cause a mild systemic illness.  Encephalitis is rare and usually associated with multiple ring-enhancing lesions
on brain imaging.

TBI in Pregnancy:
Pregnant women who develop active tuberculosis require treatment with multidrug therapy and pyridoxine. 
Treatment usually involves 3-drug therapy with isoniazid (INH), rifampin (RIF), and ethambutol for 2 months
followed by INH and RIF for 7 additional months.

Pregnant women with active TB require treatment as untreated infections carry significant risk for the mother,
fetus, and close contacts.  Treatment usually involves 3-drug therapy with isoniazid (INH), rifampin (RIF),
and ethambutol for 2 months followed by INH and RIF for 7 additional months.  All 3 of these medications
cross the placenta but are not associated with significant fetal toxicity.  Pyrazinamide, part of the 4-drug TB
treatment given to most nonpregnant individuals, is generally not administered to pregnant patients due to
uncertain teratogenic properties and little contribution to overall TB treatment efficacy.

Pregnant women undergoing treatment for TB should also receive pyridoxine (vitamin B6) supplementation to
prevent INH-induced neurotoxicity.  Counseling about medication side effects and monthly monitoring for
disease response and drug-associated hepatitis are required.

DRUGS:
INH: The most common side effect of INH is hepatotoxicity, which can occur within the first 2 months of
therapy and range from mild hepatotoxicity to hepatitis.  Risk factors include daily alcohol intake, HIV,
underlying liver disease (eg, chronic viral hepatitis), age >50, and active intravenous drug use.  Mild INH
hepatotoxicity presents with increased transaminases (usually <100 IU/L).  However, patients are usually
asymptomatic.  They typically have self-limited transaminitis and can continue INH with close monitoring.

All patients should have baseline measurement of aminotransferases before starting INH and monthly
monitoring during therapy.  Patients with normal baseline aminotransferases should discontinue the drug if
they have aminotransferases >5 times the upper limit of normal (ULN) or develop symptoms with
aminotransferases >3 times the ULN.  Patients with baseline elevated aminotransferases >3 times the ULN
should discontinue the drug if they develop symptoms (eg, jaundice, mental status changes) or 2–3-fold
increased aminotransferases from baseline.
INH can compete with vitamin B6 (pyridoxine), which acts as a cofactor in synthesizing various synaptic
neurotransmitters.  Susceptible individuals (eg, alcoholic patients, the malnourished, children, diabetic patients,
HIV patients, those with chronic kidney disease) can develop ataxia, peripheral neuropathy, and paresthesias. 
However, giving vitamin B6 along with INH (as in this patient) typically prevents neurotoxicity.

Pyrazanamide: Asymptomatic hyperuricemia is more commonly associated with the active metabolite
(pyrazinoic acid) of pyrazinamide, which competes with uric acid for elimination by the kidneys.

-Ocular toxicity is more common with ethambutol.

Cryptococcus Meningitis (5854,55,56)

Patient with a history of intravenous drug use presents with indolent manifestations of elevated intracranial
pressure including headache, blurred vision, and papilledema. The presence of extremely elevated CSF

11
pressures and molluscum contagiosum-like skin lesions raises suspicion for cryptococcal
meningoencephalitis in a patient with advanced HIV.

Cryptococcus neoformans is a yeast that commonly causes opportunistic CNS infections in patients with AIDS
(CD4 counts <100/mm3).  Manifestations typically develop over 1-2 weeks and include fever, headache, and
lethargy.  Skin findings of papular lesions with central umbilication that resemble molluscum contagiosum are
common.  CSF studies usually reveal the following:

 Markedly elevated opening pressure, often >250-300 mm H2O


 Low leukocyte count (<50/mm3) (compared to other meningitides) with a lymphocytic
predominance
 Elevated protein and low glucose
 Positive India ink preparation or cryptococcal antigen test

Patients with AIDS who have cryptococcal meningoencephalitis often have dramatic cerebrospinal fluid fungal
burdens (>1,000,000 yeast/mm3).  The yeast and capsular polysaccharides can clog the arachnoid villi, which
prevents cerebrospinal fluid outflow and increases intracranial pressure (ICP).  Pressure elevations can often be
dramatic (>250 mm H2O), resulting in headache, vomiting, visual changes, papilledema, and cranial nerve
palsies.  If untreated, brain herniation and death may occur. 

Dexamethasone is often added to the treatment of bacterial meningitis (eg, meningococcal meningitis) to
reduce inflammation, morbidity, and risk of death.  Dexamethasone is not recommended for the treatment of
cryptococcal meningitis.

Patients with AIDS who develop cryptococcal meningitis require the initiation of antiretroviral therapy (ART)
to improve CD4 count and prevent recurrence.  However, the early initiation of ART can be associated with
life-threatening immune reconstitution inflammatory syndrome; ART initiation is typically delayed until there
is sustained clinical improvement with treatment (generally after 2-10 weeks).

RX:
Patients with HIV who have cryptococcal meningitis require treatment in 3 stages as follows:

1. Induction - amphotericin B and flucytosine for >2 weeks (until symptoms abate and CSF is sterilized)
2. Consolidation - high-dose oral fluconazole for 8 weeks
3. Maintenance - lower-dose oral fluconazole for >1 year to prevent recurrence

Patients with cryptococcal meningitis who develop recurrent symptoms of elevated ICP require serial lumbar
punctures until symptoms abate.  Occasionally, recalcitrant cases require more permanent treatment such as
ventriculoperitoneal shunt.

12
D/D

-Early neurosyphilis can cause meningitis and is typically treated with aqueous crystalline penicillin G. 
Manifestations often include headache, confusion, and cranial neuropathies.  CSF lymphocyte count is usually
200-400/mm3 (unlike in this patient).

-In adults, the most common causes of bacterial meningitis are Streptococcus pneumoniae and Neisseria
meningitidis.  Empiric therapy includes a third-generation cephalosporin such as ceftriaxone in addition to
vancomycin (to cover some penicillin-resistant S pneumoniae strains).  Adults age >50 should also receive
ampicillin as they have an increased risk of Listeria monocytogenes meningitis.  In this patient, the subacute
course and low CSF leukocyte count with lymphocytic predominance make bacterial meningitis less likely.

-Cytomegalovirus can cause encephalitis in patients with advanced HIV and can be treated with ganciclovir
plus foscarnet.  Cytomegalovirus encephalitis is typically associated with altered mental status, focal
neurologic deficits, and polymorphonuclear (not lymphocytic) pleocytosis on CSF examination.

-Herpes simplex virus can cause meningitis or encephalitis and is typically treated with acyclovir.  The
absence of altered mentation makes encephalitis unlikely.  This patient with low CSF glucose and markedly
elevated opening pressure is more likely to have cryptococcal meningitis.

-Caspofungin is an echinocandin used to treat severe Candida and Aspergillus infections. 

Cutaneous Cryptococcus (6082,83)

Cryptococcus neoformans is an encapsulated yeast that primarily affects patients who are significantly
immunocompromised, particularly those with AIDS with CD4 counts <100/mm3.  Although
meningoencephalitis is the most common manifestation, pulmonary and/or disseminated disease may occur.

Cutaneous cryptococcosis is usually a manifestation of disseminated infection.  Lesions tend to arise quickly
with the head and neck preferentially affected.  Although appearance may vary, the most common form is
papules with central umbilication that resemble molluscum contagiosum. 

A small area of central hemorrhage or necrosis is an important diagnostic clue. Disseminated infections also
often affect the liver, lymph nodes, peritoneum, adrenal glands, and eyes.

DX: Diagnosis requires biopsy of a lesion; histopathological examination after staining (eg, periodic acid-
Schiff, Gömöri methenamine silver nitrate) typically reveals hyperplasia of the overlying dermis with
underlying granulomas surrounding encapsulated yeasts.

RX: Cutaneous cryptococcosis is usually treated with >2 weeks of intravenous amphotericin B and oral
flucytosine followed by a year of oral fluconazole (higher dose for 8 weeks, then maintenance).

13
D/D

-Basal cell carcinoma: Cryptococcal skin infections occasionally resemble basal cell carcinoma (pink, flesh-
colored papules).  However multiple lesions (not a single lesion) that arose quickly (not slowly), making basal
cell carcinoma unlikely.

- Disseminated Mycobacterium avium complex is a common opportunistic infection in patients with


advanced AIDS that usually manifests with fever, night sweats, abdominal pain, diarrhea, and weight loss. 
Cutaneous lesions are uncommon and are usually nodular and ulcerating.
-Kaposi sarcoma is seen primarily in men with HIV who are sexually active with other men.  Lesions are
typically red or purple papules with no necrosis.

-Pyoderma gangrenosum is a rare neutrophilic dermatitis associated with inflammatory bowel disease and
inflammatory arthritides, typically manifesting with a tender papule that degrades into a bluish, violaceous
ulcer. 

-Skin scrapings with microscopic evaluation can often diagnose fungal infection such as tinea or candidiasis. 
Cryptococcus typically requires a biopsy for cutaneous diagnosis.

Community Acquired Pneumonia (CAP)

CAP in Children (5687)


Pneumonia is a lung infection characterized by fever, tachypnea, cough, and adventitious lung sounds.  Severe
cases may be accompanied by hypoxia and respiratory distress (eg, grunting, nasal flaring, intercostal
retractions), especially in patients with a history of underlying respiratory disease (eg, asthma, chronic lung
disease in premature infants) or immunodeficiency. 

14
DX: When the diagnosis is equivocal on examination, a 2-view (posteroanterior and lateral) chest x-ray
showing infiltrates supports the diagnosis.

This child's presentation is consistent with uncomplicated community-acquired pneumonia.  The physical
examination and x-ray findings point to a lobar process, which is most commonly due to Streptococcus
pneumoniae. 

RX: The treatment of choice in the outpatient setting is high-dose oral amoxicillin, as low-dose amoxicillin is
less efficacious.

Other drug options:


-Azithromycin is the treatment of choice for atypical pneumonia (Flow chart) caused by Mycoplasma
pneumoniae but is often ineffective for S pneumoniae.

-Respiratory fluoroquinolones (eg, levofloxacin, moxifloxacin) have activity against Streptococcus and
Mycoplasma.  However, these broad-spectrum antibiotics are reserved for patients with comorbidities, as
repeated fluoroquinolone use increases the risk of fluoroquinolone resistance.

-Doxycycline is sometimes used in the treatment of atypical pneumonia caused by M pneumoniae.  It is


generally contraindicated in children age <8 due to the potential for permanent teeth staining.

Secondary pneumonia (5058)

Secondary pneumonia is a common complication of viral upper respiratory infections.  Risk is greatest in
patients of advanced age and those with underlying comorbidities (eg, chronic lung disease).  Suspicion
should be raised when patients with several days of upper respiratory infection symptoms develop worsening
fever or pulmonary manifestations (eg, productive cough, dyspnea).

Because pneumonia is a serious infection associated with significant morbidity and mortality, urgent
evaluation is required.  This patient should go to the nearest emergency department for examination,
laboratory studies, and chest x-ray.  If pneumonia is present, risk stratification with the pneumonia severity
index or CURB-65 score can help determine the appropriate care setting.  Antibiotics will likely be required.

15
Hospitalization and Drugs (5545)

Patients with community-acquired pneumonia (CAP) require risk stratification with the pneumonia severity
index or CURB-65 score to determine if hospitalization is needed.  Patients hospitalized on the medical floor
should be initiated on empiric intravenous antibiotics with an anti-pneumococcal beta lactam (eg, ceftriaxone)
and an advanced macrolide (eg, azithromycin).  This provides coverage for the most common typical (eg,
Streptococcus pneumoniae) and atypical (eg, Legionella) CAP organisms. Respiratory fluoroquinolones (eg,
levofloxacin) are also effective.

-Azithromycin monotherapy is sometimes used for CAP in previously healthy patients who are at low risk for
complications. 

- Gentamicin, piperacillin-tazobactam, and levofloxacin may be used to treat Pseudomonas aeruginosa


(combination therapy with 2 medications is required).  Although Pseudomonas often causes hospital-acquired
pneumonia.

- Vancomycin is typically added to the empiric treatment of CAP when methicillin-resistant Staphylococcus
aureus (MRSA) is suspected.  Risk of MRSA is greatest in those with recent influenza or antibiotic use,
cavitary pulmonary infiltrates, septic shock, or respiratory failure. 

16
Oseltamivir in Influenza (4946)

Influenza can be treated with antiviral medication (eg, oseltamivir) to reduce symptom duration and,
potentially, the risk of developing influenza-related complications (eg, secondary pneumonia).  However,
antiviral efficacy declines precipitously when initiated >48 hours after symptom onset.  Therefore, the Centers
for Disease Control and Prevention recommends that healthy patients with >48 hours of symptoms receive
symptomatic care (eg, acetaminophen) alone.

Patients with <48 hours of influenza symptoms should receive antiviral therapy.  In addition, patients who
require hospitalization, have severe or progressive illness, or are at high risk (eg, age >65,
immunocompromised) of influenza-related complications should be treated with antivirals regardless of
symptom duration.

Although influenza vaccination is an important preventive modality, vaccination does not guarantee immunity
to active influenza strains, and full preventative effects (humoral antibody response) may take >2 weeks to
develop.

17
CAP in AIDS (5531)

Patients with HIV are substantially more likely to develop bacterial pulmonary infections than the general
population.  The most common pathogen is Streptococcus pneumoniae, an encapsulated bacterium that is
particularly invasive in patients with HIV due to deficits in opsonization, humoral immunity, and
macrophage/neutrophil function.

Manifestations of CAP are similar between HIV-infected and HIV-uninfected individuals; patients often have
acute fever, chills, cough with sputum production, dyspnea, and pleuritic chest pain. 
DX: Laboratory studies usually show a neutrophilic leukocytosis (except when CD4 count is very low), and
chest imaging typically reveals a lobar pulmonary infiltrate.  Blood cultures are often positive, particularly
with pneumococcal pneumonia. 
RX: Patients can usually receive outpatient treatment if a single lobe is involved and respiratory status is
unaffected.

D/D

-Although risk for Legionella CAP is significantly higher in patients with HIV, this infection is much less
common than pneumococcal pneumonia.  Legionella usually causes high fever and prominent gastrointestinal
symptoms prior to the development of pulmonary symptoms.

-M TB: Patients with HIV are at much greater risk of active Mycobacterium tuberculosis infection. 
Pulmonary tuberculosis usually manifests with subacute fever, cough, weight loss, and malaise.  This patient
with acute symptoms, a cough productive of brown sputum, and a lobar infiltrate is more likely to have S
pneumoniae.

-Mycoplasma pneumoniae typically causes a subacute (not acute) illness with headache, fever, sore throat,
cough, and dyspnea.  Chest imaging typically reveals patchy interstitial infiltrates; a lobar infiltrate would be
uncommon.

-Pneumocystis pneumonia usually causes pulmonary disease in patients with CD4 counts <200/mm 3 and
develops slowly over weeks (not days).  Manifestations usually include fever, dyspnea, cough, and an
interstitial (not lobar) pulmonary infiltrate.

Note to self: NOT EVERY PNEUMONIA IN AIDS IS PCP :D

PCP (5650,5507)

18
PCP is seen in patients with defects in cell-mediated immunity.  It is most commonly seen in HIV-infected
patients with CD4 cell counts <200 cells/mm3 but can also be seen in those receiving immune modulation
following transplantation, as in this patient.

PCP has an insidious onset and usually presents with low-grade fever, nonproductive cough, dyspnea, and
tachypnea.  Hypoxia is prominent and may be out of proportion to relatively mild findings on chest
auscultation.  Prednisone may be used for immunosuppression after transplantation, and it is sometimes used
as adjunctive therapy for severe Pneumocystis pneumonia to improve air exchange. 

DX: Chest x-rays reveal diffuse, bilateral ground-glass opacities. Patients with high pretest probability for
PCP often receive empiric treatment, but definitive diagnosis requires the isolation of organisms in respiratory
secretions.  Induced sputum is the least invasive way to obtain respiratory samples; however, diagnostic
sensitivity is 50%-90%.  As such, negative results necessitate further testing if PCP is suspected.

19
RX: Trimethoprim-sulfamethoxazole is the treatment of choice for suspected or proven PCP.  Intravenous
therapy should be used initially, with oral therapy initiated once the patient has signs of clinical recovery. 

Role of Steroids in treatment (5508)


Patients with HIV who have moderate or severe Pneumocystis pneumonia (PCP) often have respiratory
decompensation during the first 2-3 days of treatment due to organism lysis, which stimulates an
inflammatory response.  To reduce the risk of intubation and death, corticosteroids are used in those with
arterial blood gas (ABG) evidence of alveolar-arterial oxygen gradient >35 mm Hg and/or arterial oxygen
tension (PaO2) <70 mm Hg (on room air).

Some research suggests that pulse oxymetry readings <92% indicate a need for corticosteroids (without
ABG).  However, ABG remains the standard of care and helps determine whether intravenous antibiotics and
hospitalization in the intensive care unit are needed.

Arterial oxygen tension <70 mm Hg on room air indicates a need for corticosteroids.

Prophylaxis: Trimethoprim-sulfamethoxazole is highly effective when used prophylactically to prevent PCP


in at-risk patients, and failure to adhere to trimethoprim-sulfamethoxazole prophylaxis is the most likely cause
of this patient's disease.

Fluconazole prophylaxis is commonly used for patients with cell-mediated immune defects to prevent
esophageal or cutaneous candidiasis.  Candida is an extremely rare cause of pneumonia.

D/D
-Influenza generally presents acutely with high fever, cough, myalgias, and pharyngitis in contrast to the more
gradual onset of PCP.
-Candida typically causes mucocutaneous infections (eg, oral thrush) in those with HIV; invasive infections
are more common in patients with central venous catheter, significant neutropenia, or hospitalization in the
intensive care unit.  Pulmonary Candida infections usually cause airspace consolidation or cavitary lesions
(not interstitial infiltrates).
-Pulmonary tuberculosis reactivation is common in those with advanced HIV and may manifest with fever,
weight loss, malaise, shortness of breath, and cough.  However, chest x-ray typically reveals cavitary infiltrates
in the upper lobes and/or mediastinal lymphadenopathy, pleural effusion, and miliary/lobar opacities.

-Staphylococcus aureus pneumonia typically causes severe, rapidly progressive, necrotizing pneumonia with
high fever, septic shock, and multilobar cavitary infiltrates.  This patient with indolent symptoms and
interstitial infiltrate is unlikely to have S aureus pneumonia.

20
-Streptococcus pneumoniae is the most common cause of community-acquired pneumonia and typically
manifests acutely (not indolently) with fever, chills, productive cough, dyspnea, and lobar (not interstitial)
infiltrate.

Aspergillosis (5921)

Aspergillus spores are frequently inhaled due to their ubiquity in the environment.  Healthy individuals easily
clear the organism and rarely develop disease, but those with severe immunosuppression (eg, prolonged
neutropenia, chronic corticosteroid use) are at risk for life-threatening infection.

Pulmonary aspergillosis is the most common manifestation and is characterized by the classic triad of cough,
pleuritic chest pain, and hemoptysis.  Chest imaging typically reveals nodules with surrounding ground-
glass opacities ("halo sign") or cavitations with air-fluid levels ("air crescent sign"). 
DX: The diagnosis is usually confirmed by serum fungal biomarkers (galactomannan and beta-D-glucan assay)
and sputum sampling for fungal staining and culture.  Bronchoscopy with bronchoalveolar lavage and/or
biopsy may be required if these diagnostic tests are inconclusive. 

RX: Treatment with intravenous voriconazole and a reduction of immunosuppressive medications is the
standard of care, but 1-year mortality is >80%.

Fluconazole is sometimes administered for Candida prophylaxis, but this antifungal agent has no activity
against Aspergillus.

D/D
-Cytomegalovirus (CMV) pneumonia usually causes fever, cough, dyspnea, and diffuse ground-glass
opacities (not a nodular infiltrate). 

21
 - Legionella can cause cough, chest pain, hemoptysis, and a nodular pulmonary infiltrate (although a lobar or
an interstitial infiltrate is more common).  However, patients usually have high fever and gastrointestinal
symptoms (eg, nausea, vomiting, diarrhea) prior to the onset of pulmonary symptoms.

-Pneumocystis pneumonia is a common infection in patients with recent stem cell transplantation and usually
manifests with diffuse interstitial pulmonary infiltrates and fulminant respiratory failure. 

-Streptococcus pneumoniae is the most common bacterial cause of pneumonia but is usually associated with a
lobar (not a nodular) pulmonary infiltrate and is less likely to occur in the setting of prophylactic levofloxacin.

Mucormycotic(5610)

Patient with diabetic ketoacidosis developed fever, periorbital swelling, and a necrotic nasal turbinate, raising
suspicion for rhino-orbital-cerebral mucormycosis.

Rhino-orbital-cerebral mucormycosis is most commonly due to the ubiquitous fungus Rhizopus oryzae.  This
organism possesses an enzyme (ketone reductase) that catalyzes growth in high-glucose, acidic environments. 
As such, >70% of patients with mucormycosis have underlying diabetes mellitus, most often with
ketoacidosis.  Manifestations include fever, nasal congestion, purulent nasal discharge, headache, and sinus
pain.  Local, necrotic spread to the palate, orbit, and brain is common.

DX: Sinus endoscopy is required for both diagnosis (biopsy and culture) and treatment (surgical
debridement).  Patients also require the prompt initiation of intravenous liposomal amphotericin B.  Once
clinically improved, amphotericin B can be transitioned to several weeks of step-down therapy with oral
antifungal medication (eg, posaconazole).

Related info:
-High-dose penicillin can treat cervicofacial Actinomyces infection, which usually manifests as a slow-
growing, painless mass near the jaw with multiple draining sinus tracts.

22
-Invasive candidiasis usually manifests with systemic symptoms and may affect a large number of organs,
including the eyes (chorioretinitis) and skin.  This patient with diabetic ketoacidosis and acute, rapidly
progressive, necrotic rhinosinusitis is more likely to have mucormycosis. Caspofungin is often used to treat
invasive candidiasis. 

-Aspergillosis is often treated with intravenous voriconazole and may occasionally present with a clinical
profile similar to that of rhino-orbital mucormycosis; however, patients are typically neutropenic, not diabetic.

-Nocardia Trimethoprim-sulfamethoxazole is often used to treat nocardiosis, which typically causes


pulmonary or central nervous system (eg, brain abscess) disease.  Sinus disease is quite rare.  Trimethoprim-
sulfamethoxazole is not an effective treatment for mucormycosis.

Candidiasis (5928)

Patient has submandibular and cervical lymphadenopathy and a white oral plaque easily removed with
scraping, suggesting oropharyngeal candidiasis (thrush).

Candida species are normal human commensals of the gastrointestinal and genitourinary tracts that may cause
mucocutaneous infections (eg, vulvovaginitis) when local microbial balance is disrupted or cellular immunity
is impaired.  Oral infections are rare in young patients but may be associated with antibiotics, inhaled
corticosteroids, or systemic chemotherapy.  In the absence of these medications (or if risk factors for HIV are
present), oropharyngeal thrush should prompt investigation for HIV infection.

The patient requires fourth-generation HIV testing (p24 antigen and HIV-1/HIV-2 antibody) and a potassium
hydroxide preparation (or Gram stain) of the mucosal scraping to confirm Candida.

Infectious Mononucleosis:

Infectious mononucleosis (IM), a disease most often due to the Epstein-Barr virus (EBV).  IM is a systemic
viral infection that is commonly seen in adolescents and young adults.  Typical presenting signs and symptoms
include fever, extreme fatigue, exudative pharyngitis or tonsillitis, lymphadenopathy (including posterior

23
cervical nodes), and hepatosplenomegaly.  Myalgias are common, and patients can develop anorexia (and
resulting weight loss), nausea, or vomiting, possibly due to hepatocellular inflammation.

The lymphocytes seen in patient's peripheral smear, known as atypical reactive lymphocytes, are a further
clue to the diagnosis.  These cells, characterized by a predominant cytoplasm with an irregular nucleus, are
commonly found with EBV infections; however, they are nonspecific and can also be seen in other systemic
viral or inflammatory conditions.

DX: The diagnostic test of choice is the Monospot test, which screens for heterophile antibodies in serum.
RX: Epstein-Barr virus infectious mononucleosis is typically a self-limited illness treated with supportive care
and nonsteroidal anti-inflammatory drugs.  Symptoms generally resolve within weeks (other than fatigue,
which persists for months). Airway obstruction may develop during the course of infectious mononucleosis,
with signs such as difficulty breathing, tachypnea, and inability to swallow.  Although corticosteroids are not
indicated in the classic self-limited cases of infectious mononucleosis, they are indicated in cases in which
severe complications (eg, airway obstruction, overwhelming infection, aplastic anemia, thrombocytopenia)
develop. Fatigue may persist for months, and women or individuals with preexisting mood disorders may be at
increased risk for prolonged fatigue.
Complication:
To prevent possible complications, a patient highly suspected of having IM should avoid all sports for a
minimum of 3 weeks since symptom onset (contact sports for a minimum of 4 weeks), regardless of physical
examination findings.

Amoxicillin administration in a patient with infectious mononucleosis classically causes a generalized


maculopapular rash.  The rash will resolve spontaneously after withdrawal of the antibiotic and observation
alone.
D/D:

-ALL & AML: Although hematologic and lymphoproliferative malignancies (eg, acute lymphocytic
leukemia, acute myeloid leukemia, Hodgkin disease) share some of the clinical features of IM (eg, fevers,
lymphadenopathy, fatigue, weight loss), exudative pharyngitis is not a common feature.  The smear would
most likely demonstrate abnormalities such as blasts (with leukemia) or Reed-Sternberg cells (with Hodgkin
disease).

24
-Parasitic infections like Malaria can present with fever, fatigue, myalgias, and hepatosplenomegaly weeks
after exposure to infection.  However, it generally results in anemia and thrombocytopenia; classically causes
cyclical fevers; does not typically cause exudative pharyngitis; and would be characterized by parasites (eg,
ring forms) on the blood smear.

Meningitis: (8867,13498,5595,5958,5959)

25
Bacterial Meningitis

This patient's clinical manifestations (fever, headache, neck stiffness) and cerebrospinal fluid (CSF) findings
(high opening pressure, neutrophilic leukocytosis, high protein, low glucose) are concerning for bacterial
meningitis. 
DX: Even though the sensitivity of CSF Gram stain varies from 60%-90%, the yield of both Gram stain and
culture may be reduced by prior antibiotic therapy. 
RX: Antibiotic pretreatment also can cause higher CSF glucose and lower protein levels in bacterial
meningitis.  Patients with high suspicion of bacterial meningitis, even with negative Gram stain and culture,
should receive a full course of antibiotic therapy.

D/D

-Viral CSF findings in aseptic viral meningitis include cell count <500/µL, >50% lymphocytes, protein
concentration <80-100 mg/dL, normal glucose concentration, and negative Gram stain.

-Cryptococcal meningitis has subacute onset of symptoms, CSF cell count of 20-200/µL, and mononuclear
cells predominance.  This patient has a negative CSF cryptococcal antigen test, which has a sensitivity of 93%-
100%.

-Encephalitis Abnormalities in brain function, including altered mental status, hemiparesis, seizures, and
aphasia, are common in herpes simplex encephalitis.  CSF analysis shows lymphocytic pleocytosis, increased
number of erythrocytes, and elevated protein.  Temporal lobe abnormalities seen on brain imaging are
considered strong evidence for herpes simplex encephalitis.

Subarachnoid Hemorrhage: Computed tomography scan is >92%-95% sensitive in diagnosing subarachnoid


hemorrhage.  If the suspicion is still high despite the negative scan, lumbar puncture is indicated to definitively

26
rule out subarachnoid hemorrhage.  CSF has a higher opening pressure and erythrocyte count, but CSF protein
and glucose are normal.  Xanthochromia confirms the diagnosis.

VIRAL Meningoencephalitis:

Fever, headache, vomiting, and neurologic findings concerning for a central nervous system infection.  In
children, enteroviruses such as coxsackievirus are the most common cause, followed by herpes simplex virus
(HSV) and arboviruses.
Meningoencephalitis is characterized by inflammation of both the meninges and the brain parenchyma. 
Nuchal rigidity and a positive Brudzinski sign, in which passive neck flexion causes involuntary flexion of the
knees and hips, are indicative of meningeal irritation. 

In contrast, encephalitis causes altered mental status (eg, confusion, behavioral changes), seizures, and diffuse
or focal neurologic abnormalities (eg, hypertonicity, hyperreflexia, weakness).
DX: A lumbar puncture is required for definitive diagnosis; however, empiric acyclovir and antibiotics
(vancomycin and a third-generation cephalosporin) should be initiated while awaiting results.  Once HSV and
bacterial infections are excluded, treatment for viral meningoencephalitis is generally supportive.

27
Dexamethasone helps prevent neurologic complications in adults with pneumococcal meningitis.  For viral
meningoencephalitis, steroids are not recommended as they do not improve outcomes.

Individuals in close contact with patients having known invasive meningococcal infection should be
prescribed prophylactic antibiotics to eradicate pharyngeal carriage of the organism.  Close contact is usually
defined either as >8 hours of exposure in close proximity (<3 feet) to the affected patient or as direct
exposure to respiratory secretions within 7 days of onset of the affected patient's symptoms.  Close contacts
include household members, roommates or intimate contacts, child care workers (such as this patient), airline
travelers seated next to the affected patient during an extended flight, and those having had direct exposure to
oral secretions.  Health care workers do not need prophylactic antibiotics unless the workers are directly
exposed to respiratory secretions (eg, mouth-to-mouth resuscitation, endotracheal intubation).

Antimicrobial chemoprophylaxis should be administered as early as possible, ideally within 24 hours of


exposure to the affected patient.  Prophylaxis given >14 days after exposure has limited efficacy and is not
recommended.  Recommended regimens include rifampin (600 mg orally twice daily for 2 days),
ciprofloxacin (500 mg single oral dose), and ceftriaxone (250 mg single dose).  All 3 regimens are effective
in decreasing the spread of meningococcal infection.

Rifampin would generally be preferred, but it interacts with oral contraceptive pills (OCPs) to decrease their
efficacy.  Rifampin increases hepatic clearance of OCPs, Ciprofloxacin and ceftriaxone do not have significant
interactions with OCPs and would be better options for this patient.

Oral rifampin (every 12 hours for a total of 4 doses) is the recommended regimen.  Ciprofloxacin and

28
intramuscular ceftriaxone can also be used.  Other antibiotics are either less effective or more difficult to
administered.

Hepatitis B
Acute Hepatitis (6072, 6073).

Acute hepatitis B virus (HBV) infection is characterized by elevated aminotransferases, positive hepatitis B
surface antigen, hepatitis B IgM core antibody, hepatitis B e antigen (indicator of high infectivity), and
detectable hepatitis B DNA. 

As patients recover from acute hepatitis B virus (HBV) infection, there is normally a steady drop in both
aminotransferase and HBV DNA levels.  Aminotransferase levels typically normalize within 2-8 weeks. 
Patients who do not clear hepatitis B surface antigen after 6 months are diagnosed with progression to chronic
HBV infection.  In addition, some patients may develop complications such as acute liver failure or
coagulopathy during the initial phase of infection.  Consequently, patients should be monitored regularly to
evaluate for the development of chronic HBV infection or hepatic decompensation.

Symptom resolution is variable and partially depends on severity of initial presentation.  Patients with a milder
onset may recover completely within days, whereas those with a more severe presentation (eg, bilirubin >10
mg/dL) may report fatigue and nausea for months following acute infection.

Post exposure prophylaxis: Hepatitis B vaccine and hepatitis B immune globulin are usually administered as
postexposure prophylaxis to health care workers exposed to contaminated blood or other body fluids.  These
vaccines should be given as soon as possible (ideally within 12-24 hours of exposure) for maximal efficacy. 

Antiviral therapy has not been demonstrated to significantly change the course of acute HBV infection, but it
may be considered for patients who have acute liver failure, immunosuppression, concomitant hepatitis C or D,
or preexisting liver disease.

29
Identification of the viral genotype is helpful in patients who develop chronic HBV infection, especially if
the patient wishes to consider interferon therapy (improved outcomes with genotype A), but it is not necessary
in acute infection.

Liver biopsy is unnecessary in this patient with acute HBV infection and whose clinical course is following an
expected pattern.  However, if the patient progresses to chronic HBV infection, a liver biopsy may be
considered to evaluate for fibrosis.

Nearly 70% of patients with acute HBV infection are asymptomatic.  The remaining 30% develop symptoms
such as anorexia, nausea, jaundice, and right upper quadrant discomfort; however, even symptomatic patients
are at very low risk of acute liver failure or other significant complications.  Therefore, unlike many forms of
acute liver injury, most cases of acute HBV (even with marked elevations in aminotransferases) resolve
spontaneously and can be managed with outpatient supportive care and close follow-up.

Hospitalization: is typically reserved for patients with severe illness, increased risk of short-term
complications, or significant comorbid conditions.  This generally includes patients with significant fever or
hemodynamic instability, impaired hepatic synthetic function (eg, abnormal coagulation markers), or other
signs of acute liver failure (eg, encephalopathy).  Hospitalization can also be considered for patients age >50,
have poor oral intake, or have minimal social support. Antiviral therapy is recommended for patients with
immunosuppression, concurrent hepatitis C, severe hepatitis, or fulminant hepatic failure.

The 3 criteria for acute liver failure are evidence of hepatic injury (eg, elevated aminotransferases),
encephalopathy, and INR ≥1.5; this patient has only 1 of these 3 criteria.  Given the very low risk of
progression to acute (fulminant) liver failure due to acute HBV infection (<1%), admission to the intensive
care unit is not warranted.

Liver biopsy is indicated for patients with unclear etiology of hepatic injury after initial evaluation or for
staging of chronic liver disease prior to treatment. 

Conversion to chronic Hepatitis (6074)


The risk of developing symptoms in acute hepatitis B virus (HBV) infection progressively increases with age
due to enhanced immune response to the virus.  As a consequence, the risk of progression from acute to
chronic HBV infection decreases with age. 

-The progression rate for perinatally acquired infection is 90% due to increased immune tolerance.
-Patients infected at age 1-5 have a 20%-50% progression rate to chronic HBV infection.

-but for adults the progression rate is under 5%

On the other hand, Hepatitis C has a much higher risk of progression to chronic infection (approximately 75%-
85%).  The acute phase is often mild or asymptomatic, and patients are typically diagnosed by serology in later
stages of the illness.  Similar to HBV infection, the severity of acute symptoms in hepatitis C infection is
inversely proportional to the risk of chronic infection.
Serum Sickness in Hep B (6121)
Serum sickness (SS) is an immune complex-mediated hypersensitivity reaction (type III) that occurs when
circulating antibodies combine with antigen (classically heterologous proteins such as equine proteins) in
blood and tissues and overload normal clearance mechanisms.  This then activates complement and causes
disease.

A minority of patients with acute hepatitis B infection develop an SS-like syndrome that is attributed to
complement activation by circulating immune complexes composed of hepatitis B surface antigen.  It is
characterized by fever, polyarthritis, and dermatitis.  Patients also frequently report fatigue or malaise.  The

30
rash of the illness is variable and can include urticaria, as seen in this patient, or several other patterns (eg,
maculopapular, petechial).  SS-like syndrome usually develops during the prodromal, preicteric phase of acute
infection and resolves with the onset of jaundice.  Other extrahepatic manifestations of hepatitis B infection
explained by circulating immune complexes include polyarteritis nodosa and glomerulonephritis (usually
membrane nephropathy and less often membranoproliferative glomerulonephritis).

D/D:
-Anaphylaxis classically involves an IgE-mediated immediate hypersensitivity reaction (type I).  Urticarial
rash occurs but arthralgias are not typical.

-Contact dermatitis is an example of T cell-mediated hypersensitivity (type IV reaction).  Skin rash occurs,
but systemic manifestations (eg, fever, arthralgias) are not typical.

Hepatitis B Vaccination(5693)

Approximately 2 billion people worldwide have been exposed to HBV.  Transmission occurs primarily from
mother to child in endemic areas (eg, India) and in those who engage in high-risk sexual practices and
intravenous drug use in lower-prevalence areas (eg, United States).  Although most infections resolve
spontaneously, approximately 5% become chronic, putting patients at risk for cirrhosis, hepatocellular
carcinoma, and transmission to others.

The Centers for Disease Control and Prevention recommends a 3-dose HBV vaccination for all infants and all
children age <19 who have not yet been vaccinated.  High-risk adults who have no evidence of prior (positive
HBV surface antibody/negative surface antigen) or current (positive HBV surface antigen) infection should
also undergo vaccination.  In the United States, this high-risk group includes people with >1 of the following:

 Multiple sexual partners or a history of a sexually transmitted disease


 Men who have sexual activity with men
 Intravenous drug users
 Patients with chronic kidney disease, hepatitis C virus, or HIV
 Household contacts of patients with chronic HBV

In addition, all pregnant women, health care personnel, and inmates who are unvaccinated (and not immune)
should also receive HBV vaccination.

31
Perinatal Hepatitis B (4962)

Infants born to mothers with active hepatitis B infection are at risk for acquiring hepatitis B either in utero or,
more commonly, during delivery due to exposure to infected genital secretions.  Maternal viral load is the
primary risk factor and directly correlates with the risk of vertical transmission to the infant.  However, with
prompt administration of hepatitis B vaccine and immunoglobulin to the infant, the risk for infection
decreases to <5%.

Infants who are vertically infected with hepatitis B generally develop chronic infection in an immune-
tolerant phase.  These infants characteristically have very high viral loads and positive HBeAg, but remain
asymptomatic and have normal or minimally elevated liver enzymes.  Therefore, liver function tests cannot be
used to detect infection. Instead, exposed infants should receive the hepatitis B vaccine and immunoglobulin
within 12 hours of birth, followed by completion of the normal hepatitis B series (at age 0, 2, and 6
months). Efficacy of immunoglobulin is markedly reduced beyond 12 hours; there is no role for
immunoglobulin at age 2 months. Serology should be obtained at the subsequent well-child visit, usually at age
9 months.  If HBsAg is not detected at that time, the infant is uninfected.

Serology should be obtained 3 months after completion of the primary hepatitis B vaccine series.  Early
serology may be falsely positive (HBsAg remains positive for up to a month after immunization, and the
immunoglobulin can persist for 6-8 months).  However, as anti-HBs decreases over time, serology obtained
after age 12 months may incorrectly identify children as vaccine non responders even if their anti-HBs levels
were appropriate at age 9-12 months.

Post exposure management of Hepatitis B (5802)

32
Patient tested positive for hepatitis B surface antigen (HBsAg), indicating active infection with the hepatitis B
virus (HBV).  HBV has a high transmission rate with percutaneous exposure, which varies depending on the
source patient's HBsAg and hepatitis B e antigen (HBeAg) status. 
-Source patients positive for both HBsAg and HBeAg have transmission rates of 40%-60%;
-Those with only HBsAg positivity have reduced transmission rates of 20%-40%. 

As a result, all health care workers (HCWs) are required to be offered hepatitis B vaccination by their
employer.  Immune response (ie, hepatitis surface antibody >10 mIU/mL) should be documented 1-2 months
after the last vaccine dose.  HCWs who do not respond to the first series of injections should be revaccinated.

HCWs with an adequate response to the vaccine (initial hepatitis surface antibody [HBsAb] titer of >10
mIU/mL) are not at risk of contracting the virus and do not require post-exposure prophylaxis even if
subsequent antibody titers fall below the threshold for immunity.  Post-exposure prophylaxis for exposed
nonimmune HCWs (unvaccinated or incomplete response to initial vaccination series) involves immediate
simultaneous administration of both hepatitis B immunoglobulin (HBIG) and HBV vaccine (at different
sites).  The HCW should then receive the remaining 2 series of HBV vaccine according to the routine
schedule.

Antiviral therapy is standard post-exposure prophylaxis for HIV, but similar therapy (eg, tenofovir disoproxil
fumarate, entecavir) does not have a role in the management of HBV exposure.

Hepatitis C (6112,6087)

Acute infection:

33
Acute hepatitis C virus (HCV) infection can develop 2−26 weeks after viral exposure.  Most patients with
acute HCV are asymptomatic.  Symptomatic patients typically develop malaise, nausea, jaundice, and right
upper quadrant pain that can last for 2−12 weeks.  Laboratory studies usually show elevated aminotransferase
levels that can be 10−20 times the upper limit.  Most patients develop anti-HCV antibodies 2−6 months after
exposure. 

DX: HCV ribonucleic acid (RNA) can generally first be detected within days to 8 weeks following viral
exposure.  This patient has negative anti-HCV antibodies, but he may not have formed anti-HCV antibodies
yet.  As a result, he requires testing for HCV RNA by polymerase chain reaction (PCR) to document acute
HCV infection.

D/D:

-Biliary disease Radiologic imaging is not needed to diagnose acute HCV infection.  Imaging can be useful in
diagnosing biliary disease (eg, choledocholithiasis), but this patient's normal alkaline phosphatase level makes
biliary disease less likely.  Abdominal ultrasonography (rather than CT) is usually the first imaging modality
used.

-Autoimmune hepatitis: Antinuclear antibodies results are helpful for suspected autoimmune hepatitis, but
this patient's IV drug use makes viral hepatitis more likely.

-IM: A Monospot test helps diagnose infectious mononucleosis, which typically presents with fatigue,
pharyngitis, lymphadenopathy, splenomegaly, and atypical lymphocytosis.  Patients can have elevated liver
function test results, although usually not to the extent seen in this patient.  In addition, this patient's history of
IV drug use makes viral hepatitis more likely.

-Hemochromatosis: Elevated serum ferritin may be helpful for confirming hemochromatosis in patients with
a history of multiple blood transfusions or a family history of hemochromatosis.

Chronic Hep C:
Hepatitis C virus (HCV) is transmitted most efficiently through percutaneous exposure to blood.  As such,
most cases in the Western world arise in those with a history of intravenous drug use.  Initial infections are
usually asymptomatic, but most lead to chronic hepatic infection, putting patients at risk for cirrhosis and
hepatocellular carcinoma.

34
Screening: For HCV is primarily done using HCV antibody testing; a positive test indicates one of the
following:

 Active, ongoing infection (chronic or acute)


 Past infection that has resolved
 False-positive result

HCV antibody testing is used for screening, as it identifies those exposed to the virus.  To determine which
patients have ongoing, active disease, further testing is required to document HCV RNA in the blood.  This
is accomplished with HCV nucleic acid testing (NAT).  Patients with positive HCV NAT are candidates for
treatment with antiviral therapy.

Chronic HCV infection is usually asymptomatic, and approximately one third of patients have normal liver
function testing.  As such, symptoms and basic laboratory assessment cannot be used to identify those with
chronic versus cleared infection

DX: The diagnosis of chronic HCV infection is a 2-step process that requires a positive serologic test result for
the HCV antibody and a confirmatory molecular test for the presence of circulating HCV RNA.  Patients with
positive HCV NAT should undergo further evaluation to identify the HCV genotype and the extent of liver
fibrosis (eg, liver biopsy); aminotransferases do not provide a clear picture of disease progression. 
Confirmation with HCV NAT is required.

Hepatitis A (6987)

Hepatitis A is typically transmitted by a fecal-oral route, and the negative anti-hepatitis A virus (anti-HAV)
antibodies in this patient make hepatitis A less likely.  This patient's negative hepatitis B surface antigen
(HBsAg) and anti-HBsAg antibodies make hepatitis B also less likely.  A window period may exist with both
negative HBsAg and anti-HBsAg antibodies, but this typically occurs after resolution of a patient's symptoms
and elevated liver function test results.

Osteomyelitis in diabetic patients (5607,5608, 5048)

Patients with poorly controlled diabetes mellitus and a large, nonhealing foot ulcer requires evaluation for
osteomyelitis. 
DX:
1. Initial screening is typically performed at the bedside with probe-to-bone testing.  Contact with the bone is
highly specific (>90%) for osteomyelitis, but sensitivity is much lower.  As such, negative probe-to-bone
testing necessitates imaging to rule out osteomyelitis.
2. Plain radiographs are often ordered first due to low cost and low radiation profile.  Evidence of cortical loss,
bony destruction, and periosteal reaction can confirm osteomyelitis, but plain radiographs have a sensitivity of
<55% and are often negative for >2 weeks after infection onset.  As such, a negative plain radiograph cannot
be relied on to definitively exclude osteomyelitis
3. The diagnostic test of choice for osteomyelitis is MRI of the foot, which has a sensitivity >90% and a high
negative predictive value.  Osteomyelitis-related bone changes are present on MRI <5 days after infection
onset; therefore, patients with symptoms for >1 week and a negative MRI are considered "ruled out" for
osteomyelitis.
4. Nuclear medicine scans, such as a 3-phase bone scan and a tagged white blood cell scan, are often used in
the diagnosis of osteomyelitis if metal hardware prevents accurate images with MRI or CT scan.  Diagnostic
sensitivities of these tests are high, but, as any cause of inflammation results in a positive test, specificity is
moderate.  MRI has the greatest overall sensitivity and stronger negative predictive value.
5. CT scans are useful in the diagnosis of osteomyelitis in patients who have contraindications (eg, pacemaker)
to MRI.  CT scans have greater sensitivity than plain radiographs but are less sensitive than MRI.

35
Pathogens: Gram-positive cocci such as Staphylococcus aureus are isolated most commonly, but
Pseudomonas aeruginosa and anaerobic bacteria (eg, Clostridium) are also frequently involved, particularly
when wounds are deep and chronic.

Positive blood cultures are far more predictive, especially in the setting of radiographic evidence of
osteomyelitis.  However, bone biopsy with culture is considered the gold standard for identifying pathologic
organisms and is usually required to determine appropriate antimicrobial therapy.

RX: Diabetic foot infections with osteomyelitis are typically treated with a combination of glucose control,
surgical debridement, weight off-loading, revascularization (if needed), and antibiotic therapy for >6 weeks. 
Serial inflammatory markers (eg, erythrocyte sedimentation rate) can help confirm treatment response.

Disseminated staph infection:

Patients with Staphylococcus aureus bacteremia often develop metastatic infections of the heart valves, lungs,
or osteoarticular structures.  Suspicion should be raised in those with continued fever, leukocytosis, or new
focal symptoms.  Vertebral osteomyelitis is a common complication of S aureus bacteremia and typically
presents with back or neck pain.  Diagnosis is generally made by MRI of the spine and is confirmed with an
open or CT-guided biopsy.

Patients with vertebral osteomyelitis often have normal plain radiographs during the first 2 weeks of infection;
therefore, this study cannot be used to rule out early osteomyelitis.  The diagnostic test of choice is MRI of the
spine, which has high sensitivity and specificity and typically detects osteomyelitis within days of initial
infection.  Confirmation then requires an open or CT-guided biopsy.

Tagged white blood cell scans are rarely used to diagnose osteomyelitis as findings are generally nonspecific. 
Other nuclear medicine tests such as PET or gallium imaging are more sensitive and specific but are typically
reserved for those with equivocal results on CT scan and plain radiograph who cannot undergo MRI.

36
Post streptococcal Pharyngitis (5765,6268,5733)

Group A streptococcal pharyngitis is the most common bacterial pharyngitis in children and typically occurs at
age 5-15.  It presents with an abrupt onset of fever, sore throat, and malaise in the absence of upper respiratory
symptoms (eg, cough, rhinorrhea, congestion).  Classic physical examination findings include tonsillar
exudates, tender anterior cervical lymphadenopathy, and palatal petechiae. 
DX: Diagnosis should be confirmed with throat culture or rapid antigen testing prior to initiation of
antibiotics.  The rapid streptococcal antigen test (RSAT) is highly specific, and antibiotic treatment can be
initiated after a positive test.  However, due to the test's poor sensitivity, throat cultures must be obtained after
all negative RSATs in children. Throat cultures are not necessary if the RSAT is positive.

Most cases of acute pharyngitis are due to viral pathogens, but 5%-15% of individuals have group A
Streptococcus (GAS) infections.  In adults, the Centor criteria score can help distinguish whether a patient is
likely to have GAS.  This clinical predictive tool assigns 1 point for each of the following manifestations:

-Tonsillar exudate
-Tender anterior cervical adenopathy
-Fever
-Absence of cough

Patients with a Centor score <3 have a low probability of a GAS infection; they do not require testing or
antimicrobial treatment and can be managed symptomatically. Patients with a Centor score of 3 (as in this
patient) or 4 are more likely to have GAS and require a streptococcal rapid antigen detection test (RADT) to
confirm the microbiologic diagnosis (recommended prior to antibiotic initiation).

The Centor criteria are most useful for identifying patients who do not need to be tested or treated for GAS
pharyngitis.  Different guidelines may use different cutoffs, but the presence of 3 criteria generally warrants
testing.

Patients who have a positive streptococcal RADT should be treated with antibiotics. 

RX: Penicillin and amoxicillin are the first-line treatment options for patients with streptococcal pharyngitis,
as these medications are safe, narrow spectrum, and cost-effective.  Penicillin is the treatment of choice for
streptococcal pharyngitis, as Streptococcus pyogenes remains very sensitive to penicillin.  The benefits of
antibiotic therapy include:

37
1. Decreased symptom severity and duration
2. Prevention of spread to close contacts
3. Prevention of suppurative complications (eg, peritonsillar abscess, cervical lymphadenitis)
4. Prevention of acute rheumatic fever

Optimal duration of oral penicillin is 10 days both to ensure full eradication of bacterial carriage and to
prevent rheumatic fever.  Daily long-term prophylactic antibiotics are indicated for patients with a history of
rheumatic fever to prevent recurrence of the disease.

Cephalosporins can be used to treat streptococcal pharyngitis in penicillin-allergic patients.  Inappropriate


prescription of these other antibiotics lead to emerging resistance of bacterial strains in general.

On the other hand, a single dose of intramuscular penicillin is bactericidal for up to a month and can be
given to patients who cannot tolerate oral antibiotics.  A 5-day course of azithromycin is acceptable alternate
therapy for penicillin-allergic patients.

Post-streptococcal glomerulonephritis typically occurs 1-3 weeks after either a cutaneous or pharyngeal
streptococcal infection.  Antibiotics are effective for primary prevention of rheumatic fever, but their role in
the prevention of post-streptococcal glomerulonephritis is unclear.

Diphtheria (5112)

Young patient who did not receive age-appropriate vaccinations has fever, sore throat, and pharyngeal grey
patches coalescing into a pseudomembrane, suggesting diphtheria infection.

Diphtheria is caused by toxigenic strains of Corynebacterium diphtheriae, a gram-positive bacillus


transmitted via respiratory secretions that largely affects children age <15.  Exotoxin released by the bacteria is
absorbed systemically and can result in dose-dependent damage to the heart, nervous system, and kidneys. 
Myocarditis occurs in up to two-thirds of patients; severe cases are associated with complete heart block and
heart failure.

Vaccination with diphtheria toxoid significantly reduces the likelihood of infection with toxigenic strains of
diphtheria; this not only reduces the risk of developing active infection but also limits asymptomatic carriage
of toxigenic strains (causing population-wide declines in prevalence).

38
D/D

-Guillain-Barré syndrome (GBS) is an acute immune-mediated neuropathy that typically occurs 2-4 weeks
after a respiratory (eg, Mycoplasma pneumoniae) or gastrointestinal (eg, Campylobacter) infection.  Morbidity
with diphtheria is toxin mediated (not immune-mediated) and is not associated with GBS.
-Hemolytic uremic syndrome causes microangiopathic hemolytic anemia, thrombocytopenia, and renal
damage.  Most cases arise in children and are associated with Shiga toxin-producing Escherichia coli. 
Diphtheria is not associated with hemolytic uremic syndrome.

-Diphtheria may cause toxin-mediated damage to the nervous system, leading to peripheral or cranial
neuropathies.  Meningoencephalitis is not associated with diphtheria.

Rheumatic Fever (5480,5481)

39
Acute rheumatic fever (ARF) is a complication of untreated streptococcal pharyngitis.  Patients and families
may not remember the initial self-resolving infection.  The diagnosis is based on the J♥NES criteria (Table). 
Two major or 1 major and 2 minor criteria are required, although the diagnosis can be made on the presence of
Sydenham chorea (SC) or carditis alone.

This patient's examination findings are classic for SC (also known as Saint Vitus dance), the most common
acquired chorea in children.  Emotional lability and decline in school performance are typically the earliest
neurologic manifestations, followed by distal hand movements that progress to facial grimacing and feet
jerking.  Movements are typically irregular and rapid.  Patients also have decreased strength throughout, and
the relaxation phase of the patellar reflex is usually delayed.  In addition, pronator drift (involuntary
hyperpronation of extended arms) is present.  The diagnosis is based on clinical presentation.

SC usually develops 1-8 months after the initial streptococcal infection; carditis and arthritis develop within
3 weeks. 
Streptococcal pharyngitis is the primary risk factor for acute rheumatic fever (ARF), and treatment with oral
penicillin is the best method of primary prevention.  Sydenham chorea (SC) is a cardinal feature of rheumatic
fever, and affected patients should receive long-acting intramuscular penicillin until adulthood for secondary
prevention, even if there is no active pharyngitis.  The goal of antibiotic therapy is to eradicate group A
streptococcus to prevent recurrent ARF and worsening rheumatic heart disease.

RX: Treatment of SC itself is primarily supportive with complete resolution occurring within months in most
patients.  Corticosteroids can reduce the duration of symptoms but are typically reserved for severe cases.

-aspirin is generally avoided in children due to the risk of Reye syndrome.

-Tetrabenazine is a dopamine antagonist used for Huntington disease but not SC.

D/D

-Huntington disease is an autosomal dominant disorder caused by a trinucleotide repeat expansion.  The
psychiatric symptoms, chorea, and dementia typically manifest at age 30-50; onset is uncommon during
childhood.

-Tardive dyskinesia is a movement disorder that occurs in patients on dopamine receptor-blocking agents (eg,
antipsychotic agents, metoclopramide).  Chorea is a side effect of these medications; this child takes no
medications.

-Tourette syndrome is characterized by involuntary motor and vocal tics that begin at age 2-15.  The physical
examination is otherwise normal, and chorea is not seen.

Lyme (5143, 5823,5824)


Lyme disease, a spirochetal illness caused by Borrelia burgdorferi, is transmitted by the deer tick Ixodes
scapularis during blood feeding. Transmission occurs when the spirochete Borrelia burgdorferi moves from
the tick gut to the tick salivary glands after 36-48 hours of attachment.  As such, a tick attached <36 hours is
unlikely to transmit Lyme disease.  If duration of attachment is unclear, tick engorgement is often used as a
rough guide; transmission risk is low if a tick is unengorged.

40
This patient had a brief tick bite and is unlikely to develop Lyme disease.  Rash often forms at the site of tick
attachment several hours after tick bite due to local cutaneous irritation from tick saliva.  This does not indicate
zoonotic transmission and requires no specific intervention or treatment.  Erythema migrans, the characteristic
spreading rash associated with Lyme disease, takes >3 days (and more commonly 7-14 days) after inoculation
to develop

Early localized Lyme disease typically occurs within 1-2 weeks of tick bite and is characterized by erythema
migrans and viral-like symptoms, including:

 Erythema migrans – a slowly spreading, erythematous rash that often starts as a confluent macule
and progresses over days into a bull's-eye shape (areas of central clearing)

Viral-like symptoms – malaise, fatigue, arthralgias, myalgias, neck stiffness, and/or headache

Most cases occur in rural parts of the upper midwestern and northeastern United States, where the tick is
endemic.  Initial transmission is often (~80%) marked by a spreading, annular rash (erythema migrans) and
systemic symptoms; however, asymptomatic or unrecognized cases often go untreated, leading to persistent
infection.

Approximately 60% of patients with untreated Lyme disease develop Lyme arthritis months or years later.  A
waxing/waning monoarticular arthritis in the knee is the most common manifestations but sometimes a few
large joints are affected at once.  In contrast to patients with forms of infectious arthritis, those with Lyme
arthritis tend to be afebrile and well-appearing and can bear weight on the affected joint.  However,
arthrocentesis is generally performed because examination findings (eg, tenderness, swelling, decreased range
of motion) can overlap with potentially destructive septic arthritis. 

41
DX: Serologic testing is often falsely negative in early infection due to an incompletely developed humoral
antibody response.  Therefore, patients with early localized Lyme disease are diagnosed clinically and treated
empirically with oral doxycycline.

In contrast to early localized disease, patients with early disseminated (eg, multiple erythema migrans lesions,
carditis, neuritis) and late (eg, arthritis, peripheral neuropathy) Lyme disease should always undergo serologic
testing prior to treatment because the humoral response against the organism is fully developed.  B burgdorferi
enzyme-linked immunosorbent assay followed by Western blot is used for diagnosis
In Lyme arthritis, the synovial fluid white blood cell (WBC) count is typically 20,000-60,000/mm³. 

RX: Intravenous ceftriaxone is the preferred therapy for severe manifestations of Lyme disease (eg, carditis
with advanced atrioventricular block, radiculopathy, meningitis).  Patients with early localized Lyme disease
and/or facial nerve palsy are typically treated with oral doxycycline.

Lyme Arthritis:
Although most cases eventually resolve after years, patients are at risk for persistent or recurrent arthritis with
joint damage.  Therefore, treatment with a 28-day course of oral doxycycline or amoxicillin is recommended
in lyme arthriris.

In the past, doxycycline was not generally used to treat Lyme disease in patients age <8 due to the risk of tooth
discoloration, but the American Academy of Pediatrics now recommends ≤21-day course of doxycycline as
first-line treatment for late Lyme disease.  Doxycycline is preferred due to better penetration into the CNS
(eradicates organisms lodged there) and excellent efficacy against coinfecting, tick-transmitted pathogens (eg,
Anaplasma phagocytophilum).  Patients are also advised to rest and avoid high-impact activities until all
symptoms resolve to reduce the risk of additional joint damage. More than 90% of patients are cured after oral
antibiotic therapy.

Intravenous antibiotics (eg, ceftriaxone) are used to treat severe complications of Lyme disease (eg, carditis,
encephalopathy); most other manifestations are treated with oral antibiotics.

D/D

Rhematic Fever: Serum antistreptolysin titers are elevated with acute rheumatic fever, which typically
presents with migratory polyarthritis rather than arthritis involving a single joint.  In addition, patients are often
febrile with an erythrocyte sedimentation rate ≥60 mm/h. Monthly antibiotic therapy with penicillin G
benzathine is recommended for patients with acute rheumatic fever, which presents as a migratory polyarthritis
after an episode of pharyngitis.

JIA: Serum anti-citrullinated peptide antibodies may be positive in aggressive cases of polyarticular juvenile
idiopathic arthritis, an inflammatory arthritis involving multiple symmetrical joints. Monoarticular joint
effusion in a the knee would be uncommon. Recurrent episodes of arthritis requiring anti-inflammatory
treatment occur in patients with juvenile idiopathic arthritis (JIA).  Oligoarticular JIA can present with knee
pain and swelling, but synovial fluid white blood cell count is usually 2,000-20,000/mm³ with a lower
predominance of neutrophils (50%-75%).  Although some aggressive forms of JIA are associated with a
positive rheumatoid factor, this patient's rheumatoid factor is normal.

-Most bacterial joint infections cause fever, severe pain, and synovial fluid leukocyte count >50,000/mm³. 
Diagnosis is made with bacterial joint fluid culture.  B burgdorferi does not grow on standard bacterial joint
culture (it requires specialized culture media that is not available in most clinical laboratories)

Lyme in pregnancy (5142)

42
Manifestations usually begin with erythema migrans, a slowly expanding erythematous rash that often
develops central clearing ("bull's-eye" appearance).  Patients may also have viral-like symptoms of fatigue,
headache, myalgias, and arthralgias.  Only 25% of patients recall a previous tick bite.

Both B burgdorferi and Treponema pallidum, the causative agent in syphilis, are spirochetal organisms.  As
such, there is theoretical concern that Lyme disease during pregnancy may be associated with an increased risk
for severe congenital fetal abnormalities and fetal demise (as seen with syphilis).  However, clinical data show
no increased risk to the fetus when the mother receives adequate treatment for Lyme disease with 14-21
days of amoxicillin or cefuroxime.

Doxycycline, which is used most commonly for the treatment of Lyme disease in nonpregnant patients, should
be avoided during pregnancy due to adverse effects on fetal long-bone development and teeth coloration.

There is no evidence to suggest that Lyme disease can be transmitted from person to person; sexual contact
and breast feeding do not increase transmission risk, and close contacts do not require chemoprophylaxis.
Rocky Mountain Spotted Fever (13572)

Rocky Mountain spotted fever (RMSF) is a tick-borne illness caused by Rickettsia rickettsii.  The classic
triad of symptoms includes fever, headache, and rash.  Early nonspecific manifestations (eg, fever, nausea,
myalgias, arthralgias) are followed by the development of erythematous macules on the wrists and ankles
around days 3-5 of illness.  Rash may spread to the trunk, palms, and soles, and macules often evolve into
petechiae.  Although initial leukocyte count is typically normal, thrombocytopenia, hyponatremia, and
transaminitis are characteristic.

DX: RMSF is diagnosed presumptively based on clinical findings as serologic testing is generally negative in
early disease. 
RX: Empiric antibiotics are initiated as soon as possible without waiting for confirmatory test results, as a
delay in treatment can lead to serious complications (eg, encephalitis, pulmonary edema, arrhythmia) and
increased mortality.  Doxycycline is the recommended antibiotic for all patients, including children and
pregnant women.  The risk of teeth discoloration is minimal with a short treatment course; no other
medications are as safe and effective.

Otitis Media (6038, 6039)

43
Child with fever, ear pain, and a bulging, erythematous, immobile tympanic membrane has the classic
presentation of acute otitis media (AOM). 
DX: The diagnosis is clinical and requires:

 Fluid (effusion) in the middle ear:  Manifesting as limited tympanic membrane motility on
insufflation
 Tympanic membrane inflammation:  Manifesting as bulging of the tympanic membrane
(sensitive and specific)

Other nonspecific signs of eardrum inflammation include otalgia, tympanic membrane erythema, and fever.

The most common bacterial pathogens are Streptococcus pneumoniae, nontypeable Haemophilus influenzae,
and Moraxella catarrhalis.  Recurrent infection within 2 weeks of treatment is sometimes caused by the same
pathogen as the initial infection due to resistant strains (as a result of AOM overdiagnosis and excessive
antibiotic prescription). 

Infection that develops >2 weeks later is usually due to different pathogens.

Concurrent otitis media and purulent conjunctivitis (otitis-conjunctivitis syndrome) is typically caused by
nontypeable H influenzae.

RX: Uncomplicated acute otitis media (AOM) should be treated empirically with a 10-day course of high-dose
amoxicillin. However, repeat infection within a month of initial treatment is concerning for beta-lactamase-
producing strains of nontypeable Haemophilus influenzae, Streptococcus pneumoniae, or Moraxella
catarrhalis.  Amoxicillin-clavulanic acid, which has the benefit of an added beta-lactamase inhibitor, is the
preferred treatment for recurrent AOM if the patient has already received amoxicillin within the same month.

44
Myringotomy and placement of tympanostomy tubes is not appropriate after a single recurrence.  If a patient
has experienced persistent treatment failure, has persistent effusion for >3 months, or has >3 episodes of AOM
in 6 months (or >4 in a year), this procedure may be considered.

Ototopical antibiotics are indicated for the treatment of otitis externa. 

Treatment failure: Tympanocentesis (ie, fluid drainage from middle ear) with Gram stain, culture, and
antibiotic susceptibility testing should be considered in children with multiple treatment failures.  However,
this procedure is invasive and unnecessary for most cases of AOM that resolve with 1 or 2 empiric antibiotic
treatment courses.

D/D
-Viral OM:Viral and bacterial causes of AOM are indistinguishable on otoscopy.  However, adenovirus
infection is a less common causative organism than nontypeable H influenzae.  In addition, adenovirus usually
causes concurrent symptoms such as nonpurulent (rather than purulent) conjunctivitis, pharyngitis, upper
respiratory infection, and gastroenteritis, which are not present in this patient.

-H influenza OM  In the past, H influenzae type b infections were a more common cause of AOM; however,
due to widespread vaccination, their incidence has decreased and most cases of H influenzae AOM are caused
by nontypeable strains.

-M catarrhalis is a common cause of AOM; however, its incidence is significantly lower than that of S
pneumoniae and H influenzae.  In addition, M catarrhalis does not cause otitis-conjunctivitis syndrome.

-Although Staphylococcus aureus is a common skin flora, it is an uncommon cause of AOM or otitis externa
except in patients with tympanostomy tubes.

Serous OM: (5104)

45
This child was treated for a recent episode of acute otitis media (fever, ear pain, red/bulging tympanic
membrane) and currently is asymptomatic, with new findings of a retracted tympanic membrane with yellow
fluid and decreased mobility.  These findings are characteristic of an effusion in the middle ear, referred to as
serous otitis media (SOM) or otitis media with effusion.  After appropriate treatment of an acute episode of
otitis media, an effusion (SOM) may persist in the middle ear for up to 3 months.

Management: If no other symptoms are present at diagnosis, watchful waiting is indicated, as spontaneous
resolution is likely to occur.  If other symptoms (eg, fever, pain, purulent effusion) are present, if effusion is
bilateral, or if effusion persists for >3 months, further therapy should be considered.

-Amoxicillin-clavulanate is indicated in cases of recurrent or resistant acute otitis media.  This patient
improved after amoxicillin therapy.  Antibiotic administration generally has no effect on the natural history of
SOM.

Audiometry is generally unnecessary unless effusion has persisted for >3 months.  The exception is high-risk
children (eg, those with developmental delays or speech disorders).

Myringotomy with insertion of tympanostomy tubes is a more invasive form of management that should be
considered only if SOM does not resolve after an appropriate period of watchful waiting.

Bacterial Sinusitis (6421)

46
Patient had symptoms of a viral upper respiratory infection and then developed 2 weeks of purulent nasal
discharge, headache, and facial pain, suggesting acute bacterial rhinosinusitis (ABRS).

Acute rhinosinusitis is the symptomatic inflammation of the nasal and sinus mucosa for <4 weeks.  The
majority of cases are due to viral upper respiratory pathogens (eg, influenza virus, rhinovirus, adenovirus) and
resolve within 10 days.  However, 2%-10% of patients develop a secondary bacterial infection, most
commonly with Streptococcus pneumoniae, Haemophilus influenzae, or Moraxella catarrhalis.

DX: Diagnosis of ABRS is typically made when >1 of the following is present:

 Persistent symptoms/signs of rhinosinusitis for >10 days


 Severe symptoms, high fever (>39 C [102.2 F]), purulent nasal discharge, and/or facial pain for >3
consecutive days
 "Double sickening" - initial improvement of viral upper respiratory symptoms for 5-6 days, followed
by clinical deterioration (eg, worsened fever, headache, nasal discharge)

RX: Patients with ABRS are usually treated with 5-7 days of oral amoxicillin-clavulanate to reduce
symptom duration.  Intranasal saline irrigation and analgesics are also often recommended

-Patients with viral rhinosinusitis can be reassured and advised to undergo symptomatic treatment (eg,
analgesics).  This patient has several features of ABRS, including persistent symptoms and "double sickening";
she requires antibiotics.

Epiglottitis (5890)

47
Unvaccinated child with acute onset of fever, respiratory distress, dysphonia, and stridor, who is drooling and
sitting forward with his neck extended (tripod, sniffing position), is the classic presentation of epiglottitis
(supraglottitis).  Epiglottitis is a bacterial infection of the upper airway resulting in inflammation and edema of
the epiglottis. 

The most common etiologic agent is Haemophilus influenzae type b, which has become less common due to
widespread vaccination; however, unvaccinated and incompletely immunized children remain at risk.

Epiglottitis is a potentially life-threatening emergency in children, as epiglottis edema can rapidly obstruct the
trachea, resulting in respiratory arrest.  Emergency airway management is the priority; therefore, preparation
for endotracheal intubation is the most appropriate next step in this patient.  This is ideally performed in the
operating room by an experienced practitioner (eg, anesthesiologist, otolaryngologist).  Intravenous antibiotics
are administered only after the airway is secure.

D/D

-Croup Racemic epinephrine and intramuscular corticosteroids are beneficial for croup
(laryngotracheobronchitis), which also can present with stridor and respiratory distress.  However, a barking
cough is a prominent feature of croup.  In addition, fever is minimal and drooling is absent, in contrast to
epiglottitis.

48
-Lateral neck radiography in epiglottitis reveals a classic "thumbprint sign" due to the edematous epiglottis. 
However, the diagnosis is usually made based on clinical presentation due to impending respiratory arrest, so
intubation should be performed prior to obtaining radiographs.

UTI (5955)
Acute, uncomplicated cystitis is common in women due to the proximity between the anus and vaginal
introitus and the short length of the female urethra.  Most cases are due to fecal flora, particularly Escherichia
coli (75%-90%).  The risk is greatest in those with recent sexual activity or a history of urinary tract infection. 

S&S: Manifestations include dysuria, urinary frequency/urgency, hematuria, and suprapubic pain.
DX: History alone (eg, phone consultation) is usually sufficient for diagnosis.  However, physical examination
and urine culture are required for those who are likely to be pregnant (eg, no contraception, no recent
menstrual period) or who likely have pyelonephritis (eg, fever, chills, flank pain) or vaginal infection (eg,
pruritus, discharge)

RX:
1. Treatment with empiric trimethoprim-sulfamethoxazole (TMP-SMX) for 3 days cures >90% of cases; this
common first-line therapy is employed in areas in which TMP-SMX resistance rates are either unknown or
<20%. 
2. Five days of nitrofurantoin is another common first-line agent.

3. Ciprofloxacin is first-line therapy for acute, uncomplicated pyelonephritis.  However, this agent is not
routinely used as a first-line therapy for acute, uncomplicated cystitis due to the risk of collateral damage (eg,
colonization with multidrug-resistant organisms) and the fact that other, narrower-spectrum medications are
highly effective.

Recurrent UTI (5420)

Recurrent urinary tract infections (UTI), defined as >2 infections in 6 months or >3 infections in 1 year. 
Risk factors for recurrent UTI include a history of cystitis at age <15 years, spermicide use, recent new sexual
partner, and postmenopausal status. 

Management: Behavioral strategies that may help decrease the risk of UTI recurrence include early postcoital
voiding, adequate oral hydration, and avoidance of spermicides. However, the most effective treatment for
recurrent cystitis is antibiotic prophylaxis.  Patients with development of a UTI temporally related to
intercourse can use postcoital antibiotic prophylaxis.  Patients with recurrent UTIs unrelated to sexual
activity are managed with daily low-dose antibiotic suppression.  Typical antibiotics used for prophylaxis
include trimethoprim-sulfamethoxazole, nitrofurantoin, cephalexin, and ciprofloxacin.

-A renal ultrasound is indicated for patients with recurrent UTIs that do not respond to antibiotics, particularly

49
when there is concern for an underlying etiology such as nephrolithiasis or obstructive uropathy.

Asymptomatic bacteriuria (5906,5369)

Asymptomatic bacteriuria is common in women, particularly as they age (prevalence is ~20% in women age
>80). 
DX: Diagnosis is made when a patient with no manifestations of UTI has a clean catch urine culture that
reveals >100,000 cfu/mL of a single organism.  Some guidelines recommend 2 clean catch urine cultures for
diagnosis; however, a second specimen typically confirms the initial result (in >80% of patients).
RX: Most cases of asymptomatic bacteriuria are transient and resolve within 2 weeks.  Repeat testing or
treatment is rarely required; patients can be reassured and observed.  Treatment is generally recommended
only for those who are pregnant, undergoing urologic procedures, or within 3 months of renal transplantation.

Other treatment options

-Seven days of oral ciprofloxacin is often used to treat acute pyelonephritis in the outpatient setting.  This
patient has no manifestations of acute pyelonephritis such as flank pain/tenderness, fever, nausea, and
vomiting.

-Even if repeat urine studies showed >100,000 cfu/mL E coli, no treatment would be required in this patient
with asymptomatic bacteriuria.  If she developed urinary tract symptoms or recurrent confusion, additional
testing or treatment could be undertaken.

In Pregnancy:

50
During pregnancy, physiologic changes such as increased urine output and compression of the bladder by the
gravid uterus often cause symptoms of urinary frequency and urgency.  However, this patient also has dysuria
and a urinalysis positive for bacteria and leukocyte esterase, a presentation consistent with acute cystitis.  The
most common pathogen for acute cystitis is Escherichia coli.  Other less common pathogens include Proteus
mirabilis and Klebsiella pneumoniae.  

Risk factors for urinary tract infection in pregnancy include the presence of asymptomatic bacteriuria,
nulliparity, pregestational diabetes mellitus, and tobacco use.

RX: Pregnant patients with clinically suspected acute cystitis are treated empirically with antibiotics for 3-7
days.  First-line antibiotics include nitrofurantoin, cephalexin, fosfomycin, and amoxicillin-clavulanate.  A
urine culture is obtained to confirm infection and to direct the modification of the antibiotic regimen should
symptoms not improve with treatment.  Because there is a high risk of persistent bacteriuria, a urine culture is
repeated for a test of cure one week after treatment completion.
Ciprofloxacin, a fluoroquinolone, is effective in treating acute cystitis in nonpregnant patients. 
Fluoroquinolones are avoided during pregnancy as they are toxic to fetal cartilage development.
Trimethoprim-sulfamethoxazole is a first-line treatment for cystitis in nonpregnant women.  However, its use
in the first trimester has been associated with neural tube defects due to the folate antagonist properties of
trimethoprim.  Use in the late third trimester is associated with neonatal kernicterus.

Pyelonephritis in pregnancy (5907)

51
Patients present with fever, nausea, flank pain, and costovertebral angle tenderness, a presentation consistent
with acute pyelonephritis.  Risk factors for acute pyelonephritis during pregnancy include age <20, nulliparity,
pregestational diabetes mellitus, sickle cell disease or trait, and tobacco use.

Complications of acute pyelonephritis in pregnancy include pulmonary edema, acute respiratory distress
syndrome, and preterm labor.  Pyelonephritis can also cause renal abscess, papillary necrosis, and renal failure.

RX: Due to the risk for complications, pyelonephritis during pregnancy requires hospitalization for
intravenous hydration and empiric treatment with broad-spectrum β-lactam antibiotics (eg, ceftriaxone,
cefepime).  Once afebrile for 48 hours, patients are placed on oral antibiotics for 10-14 days.  After treatment
completion, daily suppressive therapy (eg, low-dose nitrofurantoin or cephalexin) is initiated and is maintained
until 6 weeks postpartum to prevent recurrence. A renal ultrasound is indicated to evaluate for nephrolithiasis
or a renal abscess in patients who are still febrile despite 48-72 hours of broad-spectrum antibiotics.

UTI in children (9925)

Pyelonephritis is a kidney infection that typically presents with fever, nausea/vomiting, and flank/suprapubic
pain.  Additional findings include costovertebral angle tenderness, pyuria (eg, leukocyte esterase on dipstick
analysis or >5 WBC/hpf on microscopy), and bacteriuria.  Risk factors include female sex and a history of
bladder and bowel dysfunction, as in this girl with chronic constipation.
RX: Initial management of suspected urinary tract infections (UTIs), including pyelonephritis, includes
empiric systemic antibiotic therapy (eg, 3rd-generation cephalosporins).  Intravenous antibiotics are indicated
for patients with hemodynamic instability, the inability to tolerate oral medications (ie, persistent vomiting),
or failure of improvement on oral antibiotics.  Infants age <2 months are at increased risk of sepsis and should
also be initially treated intravenously.

52
Pediatric Constipation (9926)

Renal ultrasound is indicated in children with febrile or recurrent UTIs.  Imaging should be performed after
completion of treatment as renal inflammation during acute illness can lead to false-positive results. Persistent
fevers in a patient on oral therapy could warrant intravenous therapy.
Dysfunction of the bowel and bladder is an important risk factor for urinary tract infection (UTI) in toddlers
who are toilet training.  Children may develop abnormal elimination habits, such as stool withholding and
incomplete defecation, which lead to functional constipation.  Fecal retention causes rectal distension, which
can obstruct bladder emptying.  Incomplete, dysfunctional voiding results in stagnant urine, which promotes
microbial growth, increasing the risk for UTI.

DX: A thorough history should include assessment of dietary intake, precipitating factors (eg, toilet training),
and stool consistency and frequency. 
RX: Management includes increasing dietary fiber and water intake as well as titrating laxative use (eg,
polyethylene glycol) to produce soft, regular bowel movements.

53
Prophylactic antibiotics are often indicated in patients with vesicoureteral reflux, which is diagnosed by
voiding cystourethrogram in patients with recurrent, febrile UTIs.  Prophylactic antibiotics are not indicated
for a patient with a single, uncomplicated UTI.

Repeat urine culture may be indicated in patients with continued symptoms while on antibiotic therapy to
ensure adequate treatment of infection.  However, it is not indicated in patients with clinical resolution of a
treated UTI and does not change management.

Gonococcal infection (6138,6139)

DGI occurs when Neisseria gonorrhoeae spreads from the urogenital tract to the bloodstream.  Patients with
DGI are typically unaware of the urogenital infection and usually seek clinical attention with either mono- or
oligoarthritis or a triad of manifestations including:

 Dermatitis — 2-10 painless pustules on the distal extremities


 Tenosynovitis — swelling and pain with passive extension of multiple tendons
 Polyarthralgia — asymmetric small and large joint arthralgias

Systemic symptoms such as fever and generalized malaise are also common.

DX: Patients with suspected DGI should receive blood cultures and synovial fluid sampling (if there is an
accessible joint effusion), but these tests have low diagnostic sensitivity.  Most patients are diagnosed
presumptively when nucleic acid amplification testing of the urogenital tract is positive for N
gonorrhoeae.  Testing for other sexually transmitted infections (eg, HIV, Chlamydia) should also be
performed.

RX: Recent sexual partners of individuals diagnosed with Neisseria gonorrhoeae require evaluation and
testing regardless of the presence or absence of symptoms, as nearly 60% of gonococcal urogenital infections
are asymptomatic.  Those with uncomplicated urogenital infections should be treated with dual therapy as
follows:

54
 Intramuscular ceftriaxone — a single dose cures ~99% of infections
 Azithromycin — a single dose is added, regardless of chlamydial coinfection status, to reduce the
development and spread of cephalosporin resistance as first-line alternatives to ceftriaxone are quite
limited

These medications are typically administered simultaneously in the clinic to ensure treatment adherence.

Although a single dose of ceftriaxone is almost certain to clear the infection, the Centers for Disease Control
and Prevention recommends dual therapy for all patients with uncomplicated urogenital N gonorrhoeae
infection, regardless of chlamydia coinfection status, to limit the development of antimicrobial resistance.

C & S: Culture and sensitivity testing is usually required only in the setting of potential treatment failure (eg,
continued symptoms despite appropriate first-line treatment and no evidence of reinfection or undetected
coinfection).

Penicillin allergy: Doxycycline for 7 days can be used in place of a single dose of azithromycin for patients
who have azithromycin allergy or intolerance.  However, doxycycline monotherapy is not effective for
gonococcal urethritis due to rising rates of tetracycline resistance; patients who are given doxycycline also
require ceftriaxone.

D/D

-Lyme Arthritis seen with late Lyme disease usually involves large joints (eg, knee), is often monoarticular;
and is not generally associated with rash.  Early localized Lyme disease can present with skin rash (erythema
migrans), fever, and arthralgias; however, it is not associated with pustular lesions (as in this patient) and is
diagnosed clinically (not with antibody testing).

-Parvovirus is often asymptomatic in adults but can cause fever, arthritis/arthralgia, and a reticulated or
lacelike rash.  A pustular skin rash would be atypical.

-Herpes simplex virus (HSV) can be diagnosed by viral PCR.  HSV can cause fever but skin lesions are
typically painful and located on the genitals or oral mucosa.

Syphilis (6129,30,31)

Syphilis, a sexually transmitted infection caused by the spirochete Treponema pallidum.  Hematogenous
dissemination of the spirochete occurs early in the disease course and frequently leads to seeding of the
cerebrospinal fluid (CSF).  Although this nervous system infection is usually asymptomatic at first and may
clear spontaneously, any patient with syphilis who has neurologic symptoms (eg, headache, blurred vision)
requires a lumbar puncture to evaluate for neurosyphilis.

Patients with HIV are at greatest risk for developing neurosyphilis, particularly when CD4 counts are
<350/mm3 and rapid plasma reagin titers are >1:128. 
DX: Diagnosis is typically confirmed when CSF analysis reveals an elevated leukocyte count (>5/mm 3) and
positive VDRL or fluorescent treponemal antibody absorption (FTA-ABS) testing.

RX: Syphilis infections are characterized by periods of latency and periods of active disease.  Treatment must
be targeted to the current stage of illness, which is typically determined by:

 Symptoms
 Duration of infection (if known)

55
 Cerebrospinal fluid (CSF) evidence of neurosyphilis

Patient with positive syphilis serology & no clear evidence of primary (eg, chancre), secondary (eg, diffuse
rash, condylomata lata), or tertiary (eg, neurosyphilis, aortitis) syphilis, indicating latent infection.  As the
duration of his infection is unknown, he should be treated with intramuscular benzathine penicillin G
weekly for 3 weeks.

-Patients with primary, secondary, or early latent syphilis (<1 year duration) should receive a single dose of
intramuscular benzathine penicillin G.

-Patients with neurosyphilis or ocular syphilis require intravenous aqueous crystalline penicillin G for 10-14
days.  Although this patient has blurry vision, his ocular and CSF evaluations are normal; therefore, he does
not require treatment for ocular or neurosyphilis.

-Patients with tertiary syphilis (other than neurosyphilis) who have anaphylactic reactions to penicillin can be
treated with 2 weeks of ceftriaxone.  This patient has no evidence of tertiary syphilis and no history of
penicillin allergy.

-Patients with anaphylactic penicillin allergies who have late latent syphilis or latent syphilis of unknown
duration can receive alternate treatment with doxycycline for 28 days.  This patient has no history of penicillin
allergy; he should receive penicillin, not doxycycline, to reduce risk of treatment failure.

The Jarisch-Herxheimer reaction is an acute febrile response that often occurs after the initial treatment of
spirochetal illnesses such as syphilis.  No effective prevention is available, but manifestations are generally
self-limited and resolve within 48 hours. Most patients require no treatment, but those with severe
manifestations can be treated with an antipyretic medication (eg, acetaminophen)

56
Additional info: False positive nontreponemal testing (eg, rapid plasma reagin) can occur in patients with HIV
but is usually low titer (<1:16) and associated with negative treponemal testing (eg, FTA-ABS).  This patient
with high titer nontreponemal testing and a positive treponemal test has syphilis.  He does not require
additional treponemal testing with T pallidum enzyme immunoassay or repeat serology.  Lumbar puncture and
appropriate treatment are now required and should not be delayed due to risk of progression and spread to
others.

Syphilis in pregnancy (5019)

Treponema pallidum crosses the placenta; a syphilis infection during pregnancy can result in vertical
transmission, leading to adverse fetal outcomes including fetal growth restriction, intrauterine fetal demise,
and congenital syphilis (eg, hepatomegaly, jaundice, rash, skeletal abnormalities). 

DX: All pregnant patients are screened at the initial prenatal visit with either the rapid plasma reagin or the
VDRL test.  Repeat screening can be performed in the early third trimester for high-risk patients.  Those with a
positive screening test require confirmatory testing with a treponemal-specific test (eg, fluorescent treponemal
antibody absorption, microhemagglutination test for antibodies to T pallidum).

RX: Penicillin is the first-line therapy for the treatment of syphilis during pregnancy as it treats maternal
disease, prevents vertical transmission, and treats fetal infection.  Patients who have a penicillin allergy
require penicillin desensitization prior to treatment.  Penicillin desensitization typically involves incremental
doses of either oral or intravenous penicillin followed by treatment with intramuscular penicillin G
benzathine.  To ensure an adequate response following treatment completion, repeat serologic titers are
obtained in the third trimester (eg, 28-32 weeks gestation) and at delivery.  A 4-fold or greater decrease in the
serologic titer indicates treatment success; an inadequate response requires additional treatment.

Other Drugs:
-Azithromycin and erythromycin are second-line antibiotics for syphilis in nonpregnant patients.  Although
both are safe in pregnancy, neither crosses the placenta or treats fetal infection.

-Ceftriaxone is a second-line antibiotic for nonpregnant patients with syphilis.  However, it is insufficiently
studied for the treatment of syphilis during pregnancy.

-Doxycycline is a second-line antibiotic for nonpregnant patients with syphilis.  It is contraindicated in


pregnancy due to a risk of fetal tooth staining and abnormal fetal long bone development.

57
Neurosyphilis (11597)

History of other STDs, signs and symptoms of meningitis (headache, photophobia, neck stiffness),
accompanied by floaters, eye irritation, and hearing loss.  These features are suggestive of symptomatic
meningitis, ocular syphilis, and otosyphilis and are likely manifestations of early neurosyphilis, which occurs
most commonly during the secondary stage of syphilis (as indicated by the generalized maculopapular rash
and lymphadenopathy).

Neurosyphilis can occur at any time following Treponema pallidum infection.  The early stages usually affect
the cerebrospinal fluid (CSF), meninges, and vasculature and are not considered manifestations of tertiary
syphilis; the late forms more commonly cause disease of the spinal cord (eg, tabes dorsalis) and brain
parenchyma and are considered manifestations of tertiary syphilis.  Patients with syphilitic meningitis are
usually not as sick as those with bacterial meningitis and frequently have a prodrome several days prior to
meningitic symptoms.  A CSF-VDRL test is almost universally reactive.  Ocular syphilis can present as
impaired vision with posterior uveitis, retinitis, or optic neuritis.  Cranial neuropathies, especially of the
optic, facial, or auditory nerves, can occur.  Otosyphilis may also present with tinnitus.

D/D

HIV & IM: Both acute HIV infection and infectious mononucleosis can present with fever, lymphadenopathy,
rash, and neurologic symptoms.  However, cranial nerve palsies and visual and auditory symptoms are not
typical of acute HIV.  Neurologic symptoms of mononucleosis (eg, meningitis, cranial neuropathies) are rare
and usually develop 2-4 weeks after other acute symptoms.

-Disseminated gonococcemia presents as either a combination of rash with tenosynovitis and non-purulent
polyarthralgia or lone purulent polyarthritis.

-Meningococcal infections may present with rash and meningitis but would progress rapidly (usually over a
period of hours rather than days) to severe and life-threatening meningitis.

Chlamydia Trachomatis (5351,52)

58
Infection with Chlamydia trachomatis is frequently asymptomatic.  All pregnant patients are routinely
screened for chlamydia via nucleic acid amplification testing (NAAT) at the initial prenatal visit.  High-risk
patients (eg, age <25, multiple partners) undergo screening again in the late third trimester.  In addition,
patients with symptoms suggestive of acute cervicitis, such as postcoital spotting and mucopurulent cervical
discharge, are tested with NAAT.

NAAT is the preferred test for diagnosis of both Chlamydia trachomatis and Neisseria gonorrhoeae due to its
high sensitivity (86%-97%) and specificity (98%-99%).  NAAT false-positive and false-negative results are
rare and pregnancy does not alter the test's performance This patient had negative chlamydia testing at the
initial prenatal visit; therefore, the recent positive NAAT is due to a newly acquired chlamydia infection.

59
RX: First-line treatment for patients with chlamydia infection confirmed by nucleic acid amplification testing
is azithromycin.  All sexual partners are also treated to prevent reinfection.  Pregnant patients are retested for
chlamydia 3-4 weeks after completion of treatment to ensure response to therapy (eg, test of cure).  All patients
are retested 3 months after treatment to exclude reinfection.

Confirming response to treatment during pregnancy is important because untreated chlamydia infection can
result in obstetric complications, including preterm premature rupture of membranes, preterm labor, and
postpartum endometritis.  In addition, vertical transmission via contact between the fetus and infected maternal
discharge during delivery can result in complications, including neonatal pneumonia and neonatal
conjunctivitis, the most common infectious cause of blindness.

-Chlamydia can cause urethritis but is not a cause of upper urinary tract infections (eg, pyelonephritis).  Risk
factors for pyelonephritis in pregnancy include untreated asymptomatic bacteriuria, pregestational diabetes
mellitus, and tobacco use.

-Pathogens that cause in utero infection (eg, toxoplasmosis, Listeria, parvovirus B19) are associated with an
increased risk of fetal demise.  Chlamydia does not cause an in utero infection and is not associated with fetal
demise.

-Reactivation of herpes simplex virus infection occurs; reactivation of chlamydia infection does not.  A
positive NAAT in a patient whose test was previously negative implies recent transmission and a new
infection.

Vaginitis (6314)

60
Motile, flagellated organisms identified on a Pap test or wet mount are consistent with Trichomonas
vaginalis, a protozoan parasite that is transmitted sexually.  Clinical features in women include dysuria,
urinary frequency, dyspareunia, vaginal pruritus, and a green, frothy discharge.  Vaginal and cervical punctate
hemorrhages ("strawberry cervix") are sometimes seen.  However, vaginal trichomoniasis can also be
asymptomatic in women and identified incidentally on routine cervical cytology, as in this patient.

Irritation and exposed bleeding areas in the vagina, as well as the presence of Trichomonas, can increase rates
of HIV transmission.  As a result, screening is recommended in HIV-positive patients and in women with a
history of multiple sexually transmitted infections.
RX:  Single-dose oral metronidazole is the preferred treatment.  Therefore, regardless of symptoms, both
partners are treated with oral metronidazole to reduce the risk of reinfection and of HIV transmission.
Sexual partners should undergo clinical evaluation and testing for other sexually transmitted infections;
however, expedited partner therapy (eg, prior to evaluation) is an appropriate initial step.
To avoid infant exposure, breast milk should be expressed and discarded for 24 hours after dose
administration.

-Azithromycin is used to treat Chlamydia trachomatis, a bacterium causing a sexually transmitted infection
that typically presents as asymptomatic cervicitis.  Chlamydia is diagnosed via nucleic acid amplification
testing and is not evident on microscopy or Pap test.

Urethritis in males (5478,79)

61
Men typically develop urethritis due to sexually transmitted infections.  Manifestations usually arise 4-8 days
after inoculation and include dysuria, urethral meatus itching, and urethral discharge.  Although a number of
different organisms cause urethritis, patients are generally categorized as having gonococcal urethritis
(Neisseria gonorrhoeae) or nongonococcal urethritis (NGU) (all other pathogens).

These 2 categories can often be differentiated based on urethral fluid appearance and Gram stain.  Gonococcal
urethritis is most commonly associated with frank urethral purulence and Gram stain evidence of intracellular
gram-negative diplococci.  Patients with NGU usually have scant, watery urethral discharge and Gram stain
showing leukocytes with no organisms.

Patients with NGU usually receive empiric, in-office treatment with a single dose of azithromycin.  This
medication has high cure rates for Chlamydia trachomatis, the most common cause of NGU.  However,
treatment failure frequently occurs as azithromycin has moderate or low efficacy against other causes of NGU
such as Ureaplasma urealyticum, Mycoplasma genitalium, and Trichomonas vaginalis.

-Gonococcal urethritis is treated empirically with ceftriaxone for N gonorrhoeae and azithromycin for potential
C trachomatis coinfection (which occurs in 30% of patients with gonococcal urethritis).  This patient with an
aseptic Gram stain has NGU, not gonococcal urethritis, and should be treated with azithromycin alone.

-T vaginalis is a fairly common cause of NGU and can be treated with metronidazole.  However,
metronidazole is not used for the empiric treatment of NGU

-Epididymitis: Ofloxacin is often used to treat epididymitis caused by enteric organisms.  Patients with
epididymitis usually have pain and swelling over the epididymis.

62
Reinfection:
Patients with urethritis often have continued symptoms after initial treatment.  This can be due to reinfection,
medication noncompliance, or infection with an organism not sensitive to initial empiric treatment. 
The first diagnostic step is repeating the urethral fluid Gram stain to confirm ongoing urethritis (>2
leukocytes/hpf).  If positive, patients should undergo nucleic acid amplification (NAAT) testing of the urine
for Chlamydia trachomatis, Neisseria gonorrhoeae, Mycoplasma genitalium, and Trichomonas vaginalis (if
available).

Most cases of NGU that are not effectively treated with azithromycin are due to M genitalium infection,
which usually requires treatment with moxifloxacin.

D/D:

-Chronic prostitis A 6-week course of trimethoprim-sulfamethoxazole can be used to treat bacterial


prostatitis.  Chronic bacterial prostatitis disease usually manifests with >3 months of urinary tract symptoms
(frequency, dysuria, urgency) and/or pain in the perineum.  Acute bacterial prostatitis usually causes fevers,
chills, cloudy urine, and, sometimes, voiding symptoms.  This young, sexually active patient has had dysuria
for a few weeks and scant urethral discharge.  Urethritis is more likely, and a diagnosis of chronic prostatitis
would be premature.

Chronic Prostatitis (5484)

Chronic bacterial prostatitis is a common condition of young and middle-aged men due primarily to
intraprostatic reflux of urine with microbial seeding of the prostate.  Pathogens are similar to those that cause
cystitis and include Escherichia coli (75%-80%), Enterococcus, Klebsiella, Proteus, and Pseudomonas.  Risk
is increased in patients with a history of diabetes mellitus, smoking, and urinary tract hardware or
manipulation.

Manifestations last >3 months and typically include recurrent urinary tract infection (eg, dysuria, frequency,
peritoneal discomfort), pain in the genitourinary region, and/or pain with ejaculation.  Prostate examination is
often normal but may show hypertrophy, tenderness, or edema. 
DX: Diagnosis requires urinalysis before and after prostatic massage (or examination of expressed prostate
fluid), but patients with chronic/recurrent urogenital symptoms and bacteriuria are often treated presumptively.

RX: First-line treatment is 6 weeks of a fluoroquinolone (eg, ciprofloxacin) or trimethoprim-


sulfamethoxazole; patients may have recurrent symptoms if treated for shorter duration (as in this patient who
received a week of treatment)

Other Tests and drugs:


-A transrectal biopsy of the prostate is primarily used to evaluate for prostate cancer in patients with abnormal
prostate examinations

-Tamsulosin is an alpha-1 antagonist that relaxes the smooth muscle of the bladder neck and is often used as
first-line therapy for benign prostatic hyperplasia. 

Herpes Infection (5735,36)

Herpes zoster is due to the reactivation of varicella-zoster virus (VZV) from the dorsal root ganglia of a
sensory nerve following initial infection with varicella (chickenpox) during childhood or early adulthood. 
Risk of reactivation is greatest in those of advancing age due to waning VZV-specific cell-mediated

63
immunity.  Malignancy, liver or kidney disease, and immunocompromise (eg, chronic glucocorticoids) also
increase risk.

The rash of herpes zoster evolves quickly from erythematous papules into grouped vesicles.  The thoracic and
lumbar regions are most commonly affected, and most rashes involve a single dermatome but may extend to
immediately adjacent dermatomes.  Dermatomal pain (burning, throbbing, stabbing, pruritic) occurs in the
majority of patients and often precedes the rash by days or weeks.

DX: Diagnosis is typically made on clinical presentation alone.  Patients with localized herpes zoster who have
had lesions <72 hours typically receive antiviral treatment with oral valacyclovir to reduce transmission risk,
new lesion formation, and (possibly) risk of postherpetic neuralgia.  Analgesics for acute neuritis are also
typically required.

Disseminated infection:
Disseminated infections are characterized by the appearance of lesions outside of the primary or immediately
adjacent dermatomes.  Hospitalization for intravenous acyclovir is required due to increased risk of varicella-
related complications (eg, ocular infection).

Patients with herpes zoster may transmit varicella-zoster virus (VZV) through direct contact (primarily) or
aerosolized particles (rarely) to health care personnel (HCP) and other individuals.  As such, the Centers for
Disease Control and Prevention recommend the following precautions for hospitalized patients with herpes
zoster until lesions are crusted over (indicating they are no longer infectious):

 Localized infection - standard precautions and lesion covering


 Disseminated infection - standard precautions plus contact and airborne precautions

All HCP should also have pre-employment screening to ensure immunity to VZV; those who have not had
previous varicella infection should be immunized with a 2-dose varicella vaccine.  Only HCP with known
immunity to VZV should be allowed to interact with patients with active VZV infection; however, because
vaccination is not 100% effective, the precautions outlined above are still required.

Related info:
-Patients with herpes zoster of one dermatome or the immediately adjacent dermatomes (localized zoster)
require standard precautions and lesion covering until the lesions completely crust over. 

- Droplet precautions are typically used for illnesses transmitted via coughing and sneezing (eg, Neisseria
meningitidis, influenza).  Droplet precautions are not required for patients with herpes zoster as the virus is not
typically transmitted via the airways (transmission has rarely been linked to aerosolized virus from lesions).

-VZV polymerase chain reaction assay and viral culture are useful diagnostic tools for VZV but are typically
reserved for patients with suspected infections who have atypical skin lesions (often seen in patients with
immunocompromise). 

64
GIT Infections

Diarrhea

Travelers diarrhea (12494)

Travelers' diarrhea is common in those who travel from resource-rich to resource-limited countries, occurring
at rates as high as 50%.  The vast majority of cases are bacterial in etiology, and the leading cause is
enterotoxigenic Escherichia coli (ETEC). ETEC is transmitted via contaminated water or food and releases
enterotoxins in the intestine that cause watery diarrhea and crampy abdominal pain 1-3 days after inoculation. 
Fever, nausea, vomiting, and myalgias may also occur.  Infections are typically self-limited and resolve with
supportive therapy in 3-4 days. Travelers to developing countries can reduce their risk of ETEC infection by
avoiding peeled, uncooked fruits and vegetables; unpasteurized dairy products; food from street vendors; and
untreated water (including ice).

D/D

-Giardia lamblia is a common protozoal cause of travelers' diarrhea that can manifest with watery diarrhea,
malaise, and abdominal cramps.  However, stools are typically foul-smelling or fatty, and symptoms almost
always arise >1 week after exposure.
-Rotavirus is the most common viral cause of travelers' diarrhea and typically manifests with vomiting,
nonbloody diarrhea, and fever.  Bacterial causes are far more common, and this patient has no vomiting or
significant fever.
-Shigella is a bacterial cause of travelers' diarrhea that typically results in dysentery (bloody, mucoid diarrhea)

65
and high fever (unlike this patient).
-Vibrio parahaemolyticus is a bacterial cause of seafood-associated diarrhea that can manifest with watery or
bloody diarrhea, abdominal cramps, nausea, vomiting, and fever. 

Salmonella (5740)
Symptomatic individuals found to be infected with Salmonella enteritidis should be given replacement fluid
and electrolytes, as they are at risk for becoming dehydrated.  Since the gastroenteritis is usually self-limited
and antibiotic use has not been shown to hasten the resolution of symptoms or improve the rate of Salmonella
clearance from stool, antibiotic usage is not recommended for immunocompetent adults or children older than
one year of age.  Therefore, supportive therapy and observation would be most appropriate.
In the United States, Salmonella enteritidis is becoming increasingly resistant to ampicillin.

Preemptive therapy is warranted in patient groups at greater risk for complications.  These groups include
children younger than twelve months of age (with special attention paid to neonates) and immunocompromised
adults.  Therapy may be considered for adults at least 50 years old with known atherosclerotic disease, as they
are more prone to developing bacteremia and endovascular infection. 
RX: Effective antibiotics include ciprofloxacin, trimethoprim/sulfamethoxazole and ceftriaxone .

Salmonellosis does not need to be treated with antibiotics in immunocompetent individuals age 12 months or
older.

Giardiasis (5867)

Giardia is a ubiquitous protozoan that is responsible for diarrheal illnesses worldwide.  Transmission occurs
via the fecal-oral route or ingestion of contaminated food/water.  Exposure to untreated water during
camping or hiking is a common source of infection.  Symptoms of acute infection include bulky, greasy stools,
bloating/flatus, and weight loss. 
RX: Chronic infection leads to more severe weight loss due to malabsorption.  Symptomatic individuals
require metronidazole therapy.

Symptomatic students with positive stool samples do not need to refrain from attending school unless they are
incontinent.  With appropriate hand hygiene, continent students do not pose a significant risk of disease
transmission.  However, outbreaks often occur in institutional settings (eg, nursing homes, day care facilities)
due to poor hand hygiene or fecal incontinence.  Recreational water venues are another common source of
outbreaks, as cysts often resist chlorination and remain viable in water.  Affected individuals should therefore
refrain from attending these venues until symptoms have resolved for 2 weeks.

66
Carriers: Some exposed individuals become carriers (positive stool sample but asymptomatic) and shed cysts
in the stool for weeks to months.  These patients generally recover spontaneously and do not require
treatment.  Exceptions include those at increased risk of disease transmission (eg, infected food service
workers, diapered children in day care facilities, household contacts of immunocompromised hosts). 
Treatment of exposed, uninfected contacts is unnecessary and does not significantly affect transmission.

Food Borne (9930)

Norovirus is the most common cause of epidemic gastroenteritis and is now the most common cause of all
gastroenteritis in adults and children.  Norovirus is spread via fecal-oral contamination, and the virus can be
transmitted from food and water, fomites, or airborne droplets from vomitus.  Norovirus outbreaks are
common and occur in institutionalized settings such as nursing homes and health care facilities, in addition to
restaurants and cruise ships.

Norovirus can cause the following:

1. Asymptomatic infection
2. Fever with watery diarrhea - noninflammatory small-bowel process
3. Severe illness - fever, vomiting, headache, and other systemic symptoms

Symptoms usually start suddenly and last 48-72 hours.  Diagnosis is generally clinical, and most patients
improve with supportive care.  If needed, diagnosis can be confirmed by polymerase chain reaction or nucleic
acid-based testing.

D/D

-Campylobacter infection most commonly presents with acute onset of cramping abdominal pain and
inflammatory diarrhea (eg, mucus, blood).  Diagnosis is by stool culture.
-Clostridium perfringens is typically associated with traumatic gas gangrene but is also an important cause of
watery diarrhea.  The spores of C perfringens germinate in warm food; once ingested, the bacteria produce a
toxin in the gastrointestinal tract that causes disease.  Fever and vomiting are uncommon.

67
-Enterotoxigenic Escherichia coli is the most common causative agent of travelers' diarrhea but is much less
frequently a cause of diarrhea in outbreaks.  It classically presents with malaise, anorexia, and abdominal
cramps, followed by the acute onset of watery diarrhea.
-Listeria monocytogenes is a rare cause of epidemic gastroenteritis (usually associated with deli meats and soft
cheeses).  In addition to watery diarrhea, fever, and nausea/vomiting, L monocytogenes gastroenteritis may
also present with non-gastrointestinal symptoms such as myalgia, arthralgia, and headache.  Diagnosis can be
confirmed from special stool culture media.

Vibrio Vulnificus (12833)

V vulnificus is a free-living, gram-negative bacterium that grows in brackish coastal water and marine
environments.  Infections are acquired primarily through the consumption of raw oysters or through wound
contamination during recreational activities or raw seafood handling.  Illness tends to be mild in most patients,
but those with underlying liver disease (eg, cirrhosis, viral hepatitis) or certain chronic medical conditions (eg,
diabetes mellitus, rheumatoid arthritis) are at risk for life-threatening infection.

V vulnificus wound infections are often associated with rapidly progressive cellulitis with hemorrhagic bullous
lesions, and septic shock.  Laboratory studies usually show leukocytosis with a left shift and renal
insufficiency. 

DX: Diagnosis is made by blood and wound cultures;

RX: treatment with doxycycline and ceftriaxone should not be delayed due to the high risk of death.

D/D:

-Mycobacterium marinum is found in salt and fresh water and can cause wound infections.  However,
lesions are usually papular and ulcerative (not necrotizing and bullous) and develop over several days (not

68
hours).
-Pseudomonas skin infections are associated with hot tub exposure and usually result in folliculitis (tender
papules or nodules).
- Sporotrichosis is a subacute or chronic fungal infection that arises after skin inoculation with contaminated
organic material (eg, soil, moss).  Patients develop an ulcerative papule with similar lesions along the proximal
lymphatic chain.
-Staphylococcus aureus is a common cause of skin abscess and cellulitis.  However, this infection usually
arises over a few days (not hours) and is typically not associated with marine environments.

Clostridium Difficle (6233,8831,32)

Clostridium difficile is a gram-positive, spore-forming bacterium transmitted via the fecal-oral route. 
Nontoxigenic strains are typically benign, but toxigenic strains can cause colitis in susceptible patients (eg,
hospitalized, elderly, recent antibiotic users).

The diagnosis of C difficile–associated diarrhea (CDAD) requires:

 Characteristic symptoms – Watery diarrhea (≥3 loose stools in 24 hours) with or without lower
abdominal pain, low-grade fever, and leukocytosis
 AND
 Positive stool testing for toxigenic C difficile – Stool testing confirms the presence of toxigenic C
difficile but cannot distinguish an asymptomatic carrier from an individual with active disease

The risk of developing CDAD is increased in those with:

 Gastric acid suppression – C difficile spores are acid-resistant, but proton pump inhibitors are
thought to alter the colonic microbiome, which increases the risk of C difficile proliferation.
 Recent antibiotic use – Antibiotics disrupt the barrier function of normal colonic flora. 
Fluoroquinolones, clindamycin, and broad-spectrum penicillins/cephalosporins are most likely to
cause CDAD.
 Hospitalization – Nosocomial exposure accounts for the majority of C difficile transmissions.
 Advanced age – Older individuals often have diminished colonic immunity and greater exposure to
antibiotics, proton pump inhibitors, and hospital environments.

Minimizing antibiotic use is the most important way of reducing the risk of CDAD, but avoiding gastric acid
suppression is also an important risk-reduction strategy.

RX: Episodes are classified as nonsevere CDI, severe CDI (white blood cell count ≥15,000 cells/mm3 and/or
serum creatinine >1.5 mg/dL), or fulminant colitis (eg, hypotension, ileus, toxic megacolon).

Treatment with 10 days of oral vancomycin or fidaxomicin is generally curative, but approximately 25% of
patients develop recurrence within 30 days of antibiotic cessation.  Recurrence is marked by the reemergence
of characteristic symptoms in the presence of positive C difficile stool testing. 

In patients with fulminant disease, intravenous metronidazole is added to high-dose oral vancomycin; in those
with significant ileus, intracolonic (rather than oral) vancomycin should be considered to increase antibiotic
delivery to the colon.

Intracolonic (rather than oral) vancomycin, in addition to intravenous metronidazole, can be considered in
patients with ileus. 

69
Testing for cure of Clostridium difficile infection is not recommended, as stool studies may be positive in
asymptomatic carriers.  However, in patients with signs and symptoms of recurrence (eg, ≥3 loose stools in 24
hours, abdominal cramping, leukocytosis), repeat testing is indicated and positive results indicate active
infection.  Therefore, the presence of positive PCR alone is insufficient to diagnose CDAD recurrence in this
patient, who should be observed for now.

Management of the first recurrence includes oral vancomycin in a prolonged pulsed and tapered regimen, or
fidaxomicin.

-Chlorhexidine bathing can help prevent recurrences of methicillin-resistant Staphylococcus aureus, but it has
not been conclusively shown to reduce the risk of CDAD.

Sigmoidoscopy with biopsy is an invasive procedure and would not be indicated for a single episode of watery
stool.  Sigmoidoscopy is generally avoided in patients with CDAD due to the risk of perforation, but it is
sometimes performed to aid in diagnosing those with fulminant colitis who do not have diarrhea (these patients
usually have severe pain and are quite ill).

Drug-resistant C difficile is rare. The first recurrence of C difficile is typically treated with a prolonged course
of oral vancomycin or a 10 day course of fidaxomicin.  This patient with a single episode of loose stool should
be observed for CDAD symptoms for now.

Prevention and control of C.diff:

Clostridium difficile is the most common cause of nosocomial diarrhea.  Appropriate preventive measures
include facility infection rate monitoring, hand hygiene, contact precautions, and limiting antibiotic use.  Hand
washing with soap and water is more effective at spore elimination than alcohol-based hand sanitizers.

70
Associated Antibiotics (10090,91)

Treatment options for primary Clostridium difficile infections (CDI) include oral vancomycin or fidaxomicin. 
Risk factors include recent hospitalization, advanced age, and antibiotic use.  In patients with CDI, inciting
antibiotics should be stopped as soon as possible and patients should be switched to lower-risk antibiotics.

Imaging of the abdomen is indicated in patients with evidence of severe (white blood cell count ≥15,000/mm3
or serum creatinine >1.5 mg/dL) or fulminant (eg, septic shock, apparent ileus, megacolon) disease, or in those
with severe abdominal pain.

Intestinal Helminth (4971)

Intestinal helminths are largely a disease of the developing world where water and sewage sanitation is poor. 
In these regions, more than a billion people are infected with the most common helminths including Ascaris
lumbricoides (roundworm), Trichuris trichiura (whipworm), and Ancylostoma duodenale (hookworm).  In the
United States, most cases arise in travelers returning from a developing region or in recent immigrants.

71
Clinical manifestations typically reflect the life cycle of the infecting helminth with early, transient pulmonary
symptoms (eg, dry cough, dyspnea) followed by long-term gastrointestinal symptoms (eg, nausea, vomiting,
abdominal discomfort, diarrhea). 

DX: Peripheral eosinophilia is a defining feature, and patients often have positive fecal occult blood. 
Diagnosis is usually made with stool ova and parasite testing. 

RX: Treatment with oral albendazole is typically curative (mebendazole is slightly better for whipworm).

- Ciprofloxacin is commonly used to treat bacterial travelers' diarrhea, which usually manifests with acute
gastrointestinal symptoms (eg, diarrhea, abdominal pain, vomiting). 

-Metronidazole may be used for protozoal forms of travelers' diarrhea, including Giardia lamblia and
Entamoeba histolytica, which may present with longer (>2 weeks) incubation periods. 

-Eosinophilic gastroenteritis is an inflammatory condition of the gastrointestinal tract that may be treated with
prednisone. 

- Trimethoprim-sulfamethoxazole can be used to treat travelers' diarrhea due to enterotoxigenic Escherichia


coli, which usually manifests with acute watery diarrhea.  Peripheral eosinophilia would be atypical.

Hookworm (5083)

Hookworms are intestinal nematodes (eg, Ancylostoma duodenale, Necator americanus) prevalent in
developing tropical or subtropical nations.  Their eggs are shed into the ground, where they hatch during
periods of increased moisture.  Larvae subsequently invade human hosts, either through bare feet or by
ingestion, and migrate through the lungs (often causing a transient, nonproductive cough, as in this case) to the
pharynx, where they are swallowed and complete their maturation in the gut.  Adult worms attach to epithelial
cells in the small intestine and feed on capillary blood.

Hookworm infection is often asymptomatic.  With increasing worm burden, patients may experience
nonspecific gastrointestinal symptoms (eg, diarrhea, abdominal pain, distension) or, rarely, obstruction. 
After several months, iron deficiency anemia develops.  Eosinophilia is commonly a part of the body's
immunologic response to these organisms, so it can be a helpful in distinguishing hookworm infection from
other causes of chronic diarrhea. 

72
DX: Stool ova and parasite testing is very sensitive for this infection due to the high worm (and therefore
egg) burden in symptomatic hosts. 
RX: Antiparasitic treatment (eg, albendazole, nitazoxanide) is curative, although reinfection is common in
endemic areas.

Sporotrichosis (5202)

Sporothrix schenckii is a low-virulence, dimorphic fungus found in soil and decaying plant matter.  Humans
acquire the infection when the organism is inoculated into the skin or subcutaneous tissue.  As such,
sporotrichosis typically occurs in those who engage in outdoor vocational or recreational activities such as
landscaping or gardening.

73
The first manifestation is typically a painless papule at the site of inoculation, which soon ulcerates and drains
a nonpurulent, odorless fluid.  Over days, similar lesions usually develop along the proximal lymphatic
chain. 

DX: History and physical examination are often sufficient for diagnosis, but cultures from aspirate fluid are
typically obtained. 

RX: Treatment with 3-6 months of itraconazole is usually curative.

D/D

-Blastomycosis is a fungal infection that primarily affects the lungs (pneumonia) but may occasionally cause
verrucous skin lesions with heaped-up borders (not papular ulcerative lesions).

-Cervicofacial actinomycosis is a bacterial infection at or near the jaw.  Manifestations include a slow-
growing, nontender mass that eventually evolves into multiple abscesses, fistulas, and sinus tracks that drain a
thick, yellow, serous discharge with granules.

-Coccidioidomycosis is a fungal infection that primarily affects the lungs (community-acquired pneumonia)
and central nervous system but may occasionally cause skin manifestations.  Skin lesions are typically soft
tissue abscesses, not slowly growing papules that drain nonpurulent, odorless fluid.

-Histoplasmosis is a fungus that primarily affects the lungs but may occasionally cause a progressive
extrapulmonary infection.  Disseminated histoplasmosis may be associated with nodular, papular, or plaque-
like skin lesions, but patients usually have systemic symptoms (eg, fever, fatigue, weight loss) and are
clinically ill.

Factitious Disorder (5438)

Factitious disorder is a psychiatric condition characterized by feigning or inducing illness to assume the sick
role without obvious external benefit (eg, financial gain, avoiding work).  One of the most common ways to
induce an infection is to inject exogenous material such as sputum, urine, feces, or milk into the skin or
bloodstream.  Suspicion should be raised when a patient with no history of malignancy or immunosuppressive
condition develops recurrent polymicrobial bacteremia with varying organisms.

Factitious disorder is most common in women and healthcare workers.

Rubella (German measles) 5708

74
Congenital rubella syndrome (CRS) is a rare but debilitating complication of prenatal rubella infection,
especially if the infection occurred during the first trimester.  CRS classically presents with ocular
abnormalities, sensorineural hearing loss, and cardiac defects.  In addition to a failed hearing screen, this
patient has an absent red reflex bilaterally, suggestive of cataracts, and a continuous cardiac murmur
consistent with a patent ductus arteriosus, both classic findings of CRS.  Low birth weight and microcephaly
are also common, although brain imaging is typically normal, as in this patient.  In addition, purpuric lesions
("blueberry muffin" rash, blue/purple nonblanching nodules) may be present in a minority of cases due to
extramedullary hematopoiesis.

Prenatal screening for rubella is critical as counseling can be provided about the potential complications of
prenatal infection. 
Prevention: Administration of the measles-mumps-rubella vaccine prior to conception can prevent infection,
but the vaccine cannot be offered during pregnancy as it is a live-attenuated vaccine.  Rubella infection in
unimmunized adults may cause a transient maculopapular rash and joint pain that self-resolves.  Acute rubella
infection is treated with supportive care, including during pregnancy.

D/D

-Congenital cytomegalovirus and toxoplasmosis can also present with hepatosplenomegaly, purpuric lesions,
and sensorineural hearing loss.  Imaging, however, typically reveals periventricular calcifications in congenital
cytomegalovirus and intracerebral calcifications in toxoplasmosis.

-Congenital herpes simplex virus (HSV) occurs after primary maternal HSV infection during pregnancy and
can present with hydrops fetalis and fetal demise.  Ascending HSV infection contracted during labor and
delivery is characterized by vesicular lesions, meningitis, or disseminated disease. 

-Cong Syphilis: Although most neonates with congenital syphilis are initially asymptomatic, many develop
hepatomegaly, jaundice, and rhinitis.  A rash may be present but is typically maculopapular, not purpuric. 
Cardiac and ocular abnormalities are not seen.

75
Measles (13238)

Rubeola, also known as measles, is a highly contagious, potentially fatal, vaccine-preventable disease. 
Measles should be considered in patients with fever and a rash, especially those with a history of recent foreign
travel or incomplete immunizations, such as this patient.

Prodromal symptoms include fever and fatigue in addition to cough, coryza, and conjunctivitis.  After 2-4
days, patients are often ill-appearing with a classic maculopapular rash that starts on the face and spreads
cephalocaudally.  The rash is initially erythematous and blanching but may later coalesce and appear non-
blanching, hemorrhagic, or dark brown.  Some patients develop white, pinpoint lesions (known as Koplik
spots) on the buccal mucosa.  These lesions appear after the onset of prodromal symptoms and often resolve
when the rash appears. 

RX: Treatment is generally supportive (eg, intravenous fluids, antipyretics); however, vitamin A is indicated in
severe cases to reduce complications and mortality.

D/D

-Kawasaki disease is a vasculitis characterized by ≥5 days of fever plus ≥4 clinical criteria (conjunctivitis,
mucosal changes, lymphadenopathy >1.5 cm [0.6 in], rash, extremity changes).  This patient, with only 4 days

76
of fever and 2 clinical findings (ie, rash, conjunctivitis), does not meet the diagnostic criteria for Kawasaki
disease.

-Parvovirus causes erythema infectiosum, which presents with a nonspecific prodrome (eg, fever, cough)
followed by a rash (erythematous cheeks, reticular truncal rash) in school-aged children. 

- Streptococcus pyogenes causes scarlet fever, which presents with fever, pharyngitis, and a sandpaper-like
rash.  Cough and conjunctivitis are not associated with scarlet fever.

Catheter Related infections (8847,5047)

Central line-associated bloodstream infection (CLABSI) can prolong hospital stays and increase morbidity
and mortality.  Most cases are the result of catheter contamination with skin organisms such as coagulase-
negative staphylococci, Staphylococcus aureus, Candida species, and aerobic gram-negative bacilli.  The risk
of CLABSI is greatest when a central intravenous catheter has been left in place >6 days.

A comprehensive approach to CLABSI prevention includes the use of the following:

 A checklist prior to catheter insertion


 Proper hand hygiene prior to catheter handling
 An all-inclusive catheter kit
 Sterile barrier precautions (eg, mask and sterile gown, large drape, gloves) during insertion
 Chlorhexidine-based antiseptic for preparation of the patient's skin
 Proper catheter site care
 Catheter removal when it is no longer needed

This patient's provider should ensure that a large sterile drape is used during catheter insertion to avoid
contamination with skin organisms.

Prophylactic intravenous antibiotics are not recommended

Replacement of a catheter using a guidewire results in an increased risk of bloodstream infection.  In addition,
routine replacement of catheters is not associated with a reduced risk of CLABSI.
Hemodialysis
Patients undergoing hemodialysis through a tunneled catheter have high rates of catheter-related bloodstream
infection.  Empiric treatment typically includes vancomycin plus cefepime (or gentamicin).  Catheter removal
is indicated for those with severe sepsis, hemodynamic instability, pus from the catheter site, evidence of
metastatic infection, or symptoms that do not improve within 72 hours of antibiotic administration

77
Pediatric Sepsis (5848)

Newborns are prone to overwhelming bacterial infection due to their immature immune systems.  Fever can be
the first and only sign of a serious bacterial infection in neonates.  Symptoms of urinary tract infection,
bacteremia, and meningitis are nonspecific and can include the poor appetite and lassitude seen in this
patient.  Complete blood count may show leukocytosis.  Indirect hyperbilirubinemia is usually physiologic
in neonates, but sepsis-related hemolysis can cause persistent or worsening jaundice.  All febrile neonates
should undergo the following diagnostic tests:

 Complete blood count


 Blood culture
 Urinalysis
 Urine culture
 Cerebrospinal fluid cell count
 Cerebrospinal fluid culture

The most common causes of serious bacterial infection in children are shown in the table.  Escherichia coli
urinary tract infection accounts for most bacterial infections in neonates.  Listeria monocytogenes is
currently rare but is still recognized due to potentially devastating complications.  The recommended empiric
regimen typically consists of ampicillin plus gentamicin or cefotaxime.  Cefotaxime is preferred over
gentamicin in areas with high rates of gentamicin resistance and suspected meningitis due to better
cerebrospinal fluid penetration.

Other drugs
Ceftriaxone can displace albumin-bound bilirubin in neonates, thereby increasing the amount of free bilirubin. 
The resultant hyperbilirubinemia can cross the blood-brain barrier and increase the risk of kernicterus.

-Broad-spectrum empiric antibiotics should be administered as soon as possible.  Delaying therapy by even a
few hours in an ill-appearing infant can result in permanent morbidity (eg, brain damage) or death.  Antibiotics
can eventually be narrowed based on culture growth and sensitivity.
-Sulfamethoxazole and other sulfonamides can worsen indirect hyperbilirubinemia by displacing bilirubin
from albumin, thereby increasing the amount of free bilirubin.  Trimethoprim-sulfamethoxazole is also avoided
in patients age <6 weeks due to increased risk of methemoglobinemia.

Immunization (12720,6276)

78
Pregnant women should not receive live virus vaccines, but their household members should get all routine
immunizations on schedule

The most common side effects are low-grade fever and injection site pain.  Vaccines can be administered
during mild illness (eg, common cold) as vaccination should not worsen symptoms. 

Influenza generally occurs during winter in the United States; however, the peak of outbreaks varies year to
year and may differ by location, such as November in southern states and mid-January in northeastern states. 
In addition, sporadic cases occur early and late, so the vaccine is recommended throughout the entire flu
season (September to April)

Influenza vaccine is an inactivated intramuscular injection with no live virus component.  It is safe to
administer to household contacts of pregnant or immunocompromised patients.  In fact, pregnant women
should also receive the vaccine due to increased susceptibility to infectious complications.

The influenza vaccine contains a small amount of egg protein, but those with anaphylaxis to egg can receive
the vaccine in an outpatient or inpatient setting.  Since anaphylaxis after vaccines is rare (<1.5 per 1 million
vaccines), egg allergy (personal or family history) is not a contraindication.
The influenza vaccine is recommended for all individuals age >6 months. 

79
Varicella Vaccine (5540)

The varicella-zoster virus (VZV) vaccine is a live-attenuated vaccine that is administered typically in 2 doses:
the first at age 12–15 months and the second at age 4–6 years.  VZV vaccination is recommended in all
children without absolute contraindications (Table).  However, vaccinating household contacts of transplant
recipients is safe and recommended currently by the American Academy of Pediatrics and the Infectious
Diseases Society of America.  These organizations also recommend that pretransplant patients receive 2 doses
of the VZV vaccine at least 4 weeks prior to transplantation.  Household contacts who receive the VZV
vaccine should be monitored closely for a vaccine-associated VZV rash, which occurs in <10% of children
who receive the vaccine and is potentially contagious to immunocompromised individuals.

-If this patient develops a rash, he should be isolated from his sibling until all of the lesions have crusted over. 
Varicella immunoglobulin (VariZIG) should be administered to varicella-seronegative transplant recipients
who have been exposed to VZV.  If this patient's sister becomes exposed to VZV or a vaccine-related rash, she
should receive VariZIG as soon as possible (within 10 days of initial exposure).

80
- Live virus vaccines, including VZV, should not be administered to this patient's sister as she is status post
renal transplant and on immunosuppressive therapy.

Rabies

Human Rabies (5525)

Hydrophobia is pathognomonic for rabies; water triggers pharyngeal spasms that cause the patient to be
frightened of drinking.  The illness usually starts with a few days of nonspecific prodromal symptoms,
including fever and malaise.  Neurologic symptoms (eg, confusion, lethargy, paralysis, aphasia) develop later
in the course of the disease.

In developing countries, domestic animals (eg, dogs) are common sources of infection.  In the United States,
bats and wild animals (eg, raccoons) are the most common reservoirs of infection.  The acquisition of rabies
from bats can occur from an unrecognized bite or a scratch, and possibly by inhalation of aerosolized viral
particles.  Bats are found in all states, and spelunking (cave exploration) is a risk factor for rabies acquisition
from bats.

D/D

-Epiglottitis can cause dysphagia, odynophagia, and drooling, but affected patients typically have stridor and
rapid upper airway obstruction with illness onset.

-Histoplasmosis can occur after spelunking from exposure to bird or bat droppings.  However, respiratory
symptoms (eg, cough, chest pain, hemoptysis) are usually prominent, and hydrophobia is not seen in
histoplasmosis.

-Pertussis typically manifests as paroxysmal episodes of cough.  This patient has no cough or rhinorrhea, as is
typically seen in pertussis.

81
-Tetanus is characterized by involuntary muscle contractions but is unlikely to occur in a fully vaccinated
patient.

Rabies is an almost universally fatal disease once patients are symptomatic.  Treatment is primarily palliative.

Prophylaxis (6123)

Post-exposure prophylaxis can prevent the development of rabies infection.  There are 2 forms of prophylaxis
available:

-passive immunization with rabies immune globulin and


-active immunization with rabies vaccination. 

Most patients are diagnosed following an exposure (post-exposure prophylaxis).  However, persons whose
work/travel involves likely exposure to rabies should receive a pre-exposure prophylaxis course.

In the unvaccinated patient, post-exposure prophylaxis consists of active immunization with rabies vaccine
and passive immunization with rabies immunoglobulin.  This patient received a rabies vaccination 1 year prior
to the recent bat exposure.  For patients who have been previously vaccinated, 2 doses of active immunization
with rabies vaccine are recommended. 

Passive immunization with rabies immunoglobulin in addition to active vaccination is not recommended in
previously vaccinated individuals; this can lead to the impairment of the strength and rapidity of the immune
response.

Cat scratch Disease (11994,5144)

82
This patient's localized papule and regional lymphadenopathy following cat exposure are consistent with cat-
scratch disease (CSD).  CSD is caused by Bartonella henselae, a fastidious gram-negative bacillus carried by
the majority of cats, especially kittens.  B henselae can be transmitted by a scratch or bite, or (rarely) by a flea. 
In many cases, patients do not recall a specific scratch or bite.

Patients typically have a skin lesion that can be erythematous, papular, or nodular.  Fever is present in <50%
of cases.  Regional lymphadenopathy follows within 1-2 weeks; it is the hallmark of CSD and can be the
presenting sign if the skin lesion goes unnoticed.  Affected lymph nodes are tender and erythematous;
suppuration is less common. Since bites and scratches usually occur on the extremities, ipsilateral axillary or
inguinal node involvement is most common.  However, when the inoculation is near the face, adenopathy of
the preauricular or cervical nodes is seen.  Involvement of the conjunctivae (as in this case) is an uncommon
but characteristic presentation of CSD.

Oculoglandular syndrome is a specific presentation of cat-scratch disease characterized by conjunctivitis and


ipsilateral preauricular and cervical adenopathy.  The most common complication of cat scratch disease is
lymph node suppuration.

DX: Cat exposure and symptoms consistent with CSD are usually sufficient to make a clinical diagnosis. 

RX: Mild cases in healthy patients often self-resolve within 1-4 months; however, azithromycin is effective
against B henselae and has been shown to decrease the length and severity of symptoms. 

In unclear cases, antistaphylococcal and streptococcal coverage (eg, clindamycin) may be added to empirically
treat the most common organisms causing lymphadenitis.

Human Bite (5415,16)

83
Human bites can be complicated by a polymicrobial soft-tissue infection.  Eikenella corrodens, a gram-
negative anaerobe, is a common finding in infections from human bites.  As in animal bites, management of
human bites includes local wound care, antibiotics, and consideration of tetanus prophylaxis.

RX: Treatment must include anaerobic coverage, particularly against Eikenella corrodens.  Oral
amoxicillin/clavulanate (or ampicillin-sulbactam if intravenous therapy is indicated) is the drug of choice for
such infections as it covers all of the common pathogens in human bite infections.

Wound care (5147)

Mammalian bites can lead to infection from either oral flora of the animal (eg, Pasteurella, oral anaerobes)
or skin flora of the patient (eg, Streptococcus pyogenes, Staphylococcus aureus).  Cat bites are considered
particularly high-risk as feline teeth are thin and sharp, resulting in puncture wounds that inoculate bacteria
deep into tissue.  Bites also carry a higher risk of infection if they are on the hands/feet, >12 hours old, and in
immunocompromised patients.

For patients with any of these historical features, wounds should be left open to heal by secondary intention
and antibiotic prophylaxis should be provided. 

RX: Amoxicillin-clavulanate is the first-line agent for prophylaxis following mammalian bites as it provides
coverage against both gram-negative (eg, Pasteurella) and gram-positive (eg, S pyogenes) organisms as well as

84
oral anaerobes.

Tetanus (5688)

Following a mammalian bite, the decision to administer tetanus prophylaxis (ie, tetanus toxoid-containing
vaccine ± tetanus immune globulin) is based on the severity of the wound and the immunization status of the
patient.

 For patients who have received ≥3 tetanus toxoid doses, a booster dose of tetanus toxoid is indicated
only if their last tetanus dose was ≥10 years ago (for clean or minor wounds) or ≥5 years ago (for
dirty or severe wounds). 
 Patients who have received <3 tetanus toxoid doses (ie, incompletely immunized) or whose vaccine
status is uncertain should receive a tetanus toxoid booster.  If such patients have dirty or severe
wounds, they should also receive tetanus immune globulin.

-Prompt cleansing of bite wounds is critical in infection prevention but does not preclude the need for tetanus
prophylaxis for at-risk patients.  This patient does not require a tetanus toxoid booster only because he is
completely immunized and his last dose was <5 years ago.

-The availability of the animals can impact the decision to administer rabies post-exposure prophylaxis but
does not factor into the decision to give tetanus prophylaxis.  Tetanus prophylaxis decisions are based on the
severity of the wound and immunization status.

Endocarditis (5544)

Intravenous drug users are at increased risk of developing IE, and HIV infection further increases the risk,
especially with low CD4 lymphocyte count.  Although left-sided IE is more common in the general population,
intravenous drug users most commonly develop right-sided IE affecting the tricuspid valve.  The propensity
for right-sided IE may relate to injected particulate matter causing damage to the tricuspid valve (allowing for
bacterial adherence) before being filtered out by the pulmonary capillaries (therefore never reaching the mitral
or aortic valve).

Staphylococcus aureus is responsible for >50% of cases of IE in intravenous drug users; the presentation is
typically acute and characterized by high fever.  Patients often have pleuritic chest pain and cough due to
septic pulmonary emboli, a complication that occurs in 75% of cases of right-sided IE.  The emboli appear on

85
chest x-ray as multiple nodular opacities that may have evidence of cavitation.  Cardiac murmur is
sometimes absent in right-sided IE, and peripheral signs of IE (eg, splinter hemorrhages, Janeway lesions) are
usually absent.  The painful subcutaneous nodule in this patient's antecubital region represents septic
thrombophlebitis (a clue to intravenous drug use) rather than a peripheral manifestation of IE.

-Urine antigen testing is useful in diagnosing Legionella pneumonia.  The infection is often characterized by
high fever; however, accompanying gastrointestinal symptoms (eg, vomiting, diarrhea) are often present and
chest x-ray typically reveals unilobar consolidation rather than multiple discrete opacities.

Infective Endocarditis (5237)

Clinical presentation, with fever, petechiae, history of intravenous drug abuse, and holosystolic murmur at the
apex, is suggestive of infective endocarditis (IE) involving the mitral valve.  The headache, lethargy, and neck
stiffness suggest subarachnoid hemorrhage secondary to rupture of a mycotic aneurysm.  

Mycotic or infected arterial aneurysms can develop due to metastatic infection from IE, with septic
embolization and localized vessel wall destruction in the cerebral (or systemic) circulation.  Intracerebral
mycotic aneurysms can present as an expanding mass with focal neurologic findings or may not be apparent
until aneurysm rupture with stroke or subarachnoid hemorrhage. 

DX: The diagnosis of mycotic cerebral aneurysm can usually be confirmed with computed tomography
angiography. 

RX: Management includes broad-spectrum antibiotics (tailored to blood culture results) and surgical
intervention (open or endovascular).

-Patients with basilar artery occlusion present with motor weakness, ataxia or incoordination, altered level of
consciousness, facial weakness, dysphagia and/or dysarthria, and unilateral or bilateral gaze paralysis.  The
presence of neck stiffness is more suggestive of subarachnoid hemorrhage.

-Progressive multifocal leukoencephalopathy is a demyelinating illness of the central nervous system that
typically occurs in immunosuppressed patients, especially those with AIDS.  It is caused by reactivation of the
polyomavirus JC (JC virus) and presents with neurologic deficits including hemiparesis, gait ataxia, visual
symptoms, and altered mental status.  It is not seen in non-immunosuppressed patients, and fever is not typical.

86
Ecthyma gangrenosum (6052)

Ecthyma gangrenosum can be due to a number of different pathogens but is most commonly associated with
Pseudomonas aeruginosa bacteremia in patients with immunocompromise (eg, neutropenia,
immunosuppressant medications).  Bacteria invade the perivascular structures (media, adventitia) of veins or
arteries and induce secondary ischemic necrosis of the overlying tissue.  This manifests with painless, red
macules that rapidly progress to pustules/bullae and then quickly evolve into gangrenous ulcers with raised
violaceous margins.  The anogenital area, axilla, and extremities are most commonly affected, and single or
multiple lesions may occur.  Fever and signs of systemic illness are typically present.

DX: This is a life-threatening infection; the patient should receive blood and wound cultures and then be
initiated on empiric intravenous antibiotics.

RX: Common regimens include an antipseudomonal beta-lactam (eg, piperacillin-tazobactam) and


aminoglycoside (eg, gentamicin).  Selected cephalosporins, monobactams, fluoroquinolones, and carbapenems
may also be used.  Patients are typically transitioned to monotherapy once culture results and antibiotic
sensitivities are available.
D/D

-Clostridial myonecrosis, also called gas gangrene, is associated with fever, severe muscle pain, and purple-
colored bullae. 

-Candida Patients with neutropenia and central venous access are at greater risk for invasive Candida
infections.  Skin lesions may occur but typically manifest as clusters of painless pustules on an erythematous
base, sometimes with necrotic centers. 
-Mycosis fungoides is a cutaneous T-cell lymphoma that typically causes slowly (not rapidly) progressive skin

87
lesions that appear as patches, plaques, tumors, or erythroderma.
-Pyoderma gangrenosum is a neutrophilic dermatosis that is usually associated with inflammatory bowel
disease or an arthritides.  Lesions typically begin as inflammatory nodules, pustules, or vesicles and quickly
evolve to ulcers; however, they are usually quite painful.

Lactation Mastitis (9962,63,64)

Women are at greatest risk of developing mastitis during the first 3 months postpartum, when breastfeeding is
still becoming established.  Difficulties with breastfeeding (eg, poor latching) can lead to prolonged
engorgement, inadequate milk drainage, and clogged milk ducts (tender, palpable lump or "cord").  Bacteria
from the skin can enter the milk ducts and proliferate in stagnant milk, resulting in infection. 

The most common organism is Staphylococcus aureus. 

DX: Diagnosis is clinical and no further workup is indicated. 

RX: First-line antibiotics include oral dicloxacillin (antistaphylococcal penicillin) and cephalexin (first-
generation cephalosporin); both are safe during breastfeeding.

88
-Intravenous vancomycin is the treatment of choice for suspected methicillin-resistant S aureus (MRSA).  It is
indicated in the treatment of lactational mastitis in patients who have MRSA risk factors (eg, recurrent
hospitalizations/antibiotics, incarceration), hemodynamic instability, or failed outpatient therapy.

Cold compresses, nonsteroidal anti-inflammatory medication, and breast pumping are first-line treatments of
breast engorgement, which typically presents with diffuse, bilateral breast tenderness.

Breast pain and mastitis are common reasons for early discontinuation of breastfeeding.  Pain during
breastfeeding is frequently due to poor positioning or latching, which can be evaluated by a lactation
consultant.

Breast Abscess

Breast abscess development is an uncommon complication of lactational mastitis but should be suspected in
patients whose symptoms do not improve or worsen despite antibiotic therapy.  The characteristic finding of an
abscess is a focal area of fluctuance (wavelike motion on palpation, indicating pus collection).  Induration
(skin thickening from edema and inflammation) can occur in both mastitis and abscess. 
DX: Ultrasound is the gold-standard modality for differentiating these conditions; it is quick, safe,
inexpensive, and widely available, and it diagnoses abscess more accurately than physical examination.  If an
abscess is identified, ultrasound is also used to guide fine-needle aspiration under local anesthesia.

RX: Incision with drainage is the preferred treatment for most skin abscesses.  However, this treatment for a
breast abscess has increased risk of complications (eg, milk fistulas), slower recovery time, and a less desirable
cosmetic outcome.  Therefore, incision and drainage are reserved for abscesses that do not improve with
antibiotics and fine-needle aspiration.

89
Malaria (11848)

The constellation of fevers, headaches, and laboratory findings with thrombocytopenia raises suspicion for
malaria, specifically infection with Plasmodium falciparum. 

DX: A peripheral blood smear is useful in establishing the diagnosis.  Antimalarial prophylaxis, which
generally needs to be started in advance of a trip and continued for some time after a traveler's return, helps
minimize the risk of infection.  The specific regimen depends on the patient and parasite resistance patterns at
the travel destination; the most recent guidelines are updated on the Centers for Disease Control and
Prevention website.

The differential diagnosis for fever in a returning traveler depends on geographic epidemiology and is often
divided into 3 categories (with some overlap):

 Early incubation period (<10 days) infections include typhoid fever, dengue fever, chikungunya,
influenza, and legionellosis
 Medium incubation period (1-3 weeks) infections include malaria, typhoid fever, leptospirosis,
schistosomiasis, and rickettsial disease

Longer incubation period (>3 weeks) infections include tuberculosis, leishmaniasis, and enteric parasitic
infections.

90
Dengue Fever (11394)

Patient's febrile illness after recent travel to an endemic country suggests dengue fever (DF), which is due to
viral infection spread by the Aedes mosquito.  DF is prevalent in tropical and subtropical regions (South and
Southeast Asia, Pacific Islands, Caribbean, Americas).  It can present as an acute febrile illness with headache,
retro-orbital pain, joint/muscle pain, macular rash, cervical lymphadenopathy, and pharyngeal erythema.  Other
findings can include hemorrhagic tendencies (eg, petechiae with tourniquet application) or spontaneous
hemorrhage (eg, purpura, ecchymoses), thrombocytopenia, leukopenia, and elevated liver aminotransferases.

Dengue hemorrhagic fever (DHF) is the most serious dengue viral infection and is due to increased capillary
permeability leading to hemoconcentration, pleural effusion, and ascites.  Circulatory failure can develop
with significant plasma leakage and is sometimes referred to as dengue shock syndrome.  Patients typically
have marked thrombocytopenia (<100,000/mm3) and prolonged fever.

91
- Cerebral edema can occur in Plasmodium falciparum malaria, which usually presents with fever, anemia, and
splenomegaly.  This patient's absence of anemia makes this less likely.  In addition, this patient took all
advised prophylactic medications, which included malaria prophylaxis for travel to India.

-Intestinal perforation can be a complication of typhoid fever, which typically presents more gradually with
rising fever, chills, relative bradycardia, abdominal pain, and rose spots (faint salmon-colored macules on the
trunk and abdomen).

-Leptospirosis can sometimes progress to Weil's disease with renal failure and jaundice.  The lack of the
characteristic conjunctival suffusion makes leptospirosis less likely.

Chagas Disease (9869)

The natural history of Chagas disease has 3 distinct phases. 

1. The first is an acute phase of mild, nonspecific symptoms with fever or myalgias,
2. An indeterminate phase with serologic or parasitic evidence of T cruzi infection in the absence of signs or
symptoms of infection. 
3. The final phase is chronic Chagas cardiomyopathy and gastrointestinal disease.

Chronic Chagas cardiomyopathy is the leading cause of dilated cardiomyopathy in Central and South
America.  Patients can remain asymptomatic for several years or develop symptoms of heart failure.  Findings
of right-sided heart failure (eg, jugular venous distension, ascites, edema) are more pronounced than those of
left heart failure (eg, dyspnea, crackles).  These patients frequently develop arrhythmias, including sinus node
dysfunction, complete heart block, and ventricular tachycardia. 

DX: Echocardiogram shows varying degrees of biventricular dysfunction, along with left ventricular apical
aneurysm, which is considered pathognomonic of Chagas cardiomyopathy.

-Borreliosis or Lyme disease is a tick-borne illness caused by an infection with the spirochete Borrelia
burgdorferi.  Lyme carditis usually manifests as varying degrees of atrioventricular conduction block.  In rare
cases, patients can also develop myopericarditis with dilated cardiomyopathy.  However, apical aneurysms are
not typically seen in patients with Lyme carditis.

-Giant cell myocarditis is a rare form of idiopathic myocarditis thought to be mediated by an autoimmune
process.  Depending on severity, the echocardiographic findings in patients with myocarditis can include left
ventricular dilation, with segmental and/or global left ventricular systolic dysfunction.

-Infection with human immunodeficiency virus has been associated with the development of dilated
cardiomyopathy.  Left ventricular apical aneurysm does not occur in patients with HIV-associated
cardiomyopathy.

West Nile virus (4967)

Viral infections of the central nervous system (CNS) can cause inflammation of the meninges (ie, meningitis),
the brain parenchyma (ie, encephalitis), or both (ie, meningoencephalitis).  This patient has nuchal rigidity
(meningeal sign) as well as altered mental status (encephalopathy), consistent with viral
meningoencephalitis.  In children, enteroviruses (eg, Coxsackievirus, Echovirus) are the most common cause
of viral CNS infections, followed by herpesviruses and arboviruses.

92
West Nile virus is an arbovirus transmitted by mosquitoes, causing infection in the summer and early fall, as
seen in this patient.  Patients with neuroinvasive disease typically present with manifestations of meningitis,
including fever, headache, photophobia, nuchal rigidity, and/or signs of encephalitis, such as confusion and
focal neurologic findings (eg, hyperreflexia).  A maculopapular rash is common with West Nile virus but not
always present, and patients may have excoriated mosquito bites, as seen on this patient. 
DX: Detection of West Nile IgM antibody in the cerebrospinal fluid is diagnostic, and treatment is generally
supportive.

D/D

-Lyme Typical CNS manifestations include meningitis, cranial nerve palsies, and peripheral neuropathies;
encephalitis is rare.

-Cytomegalovirus infection of the CNS occurs almost exclusively in HIV-infected patients with AIDS or
post-transplant patients.

-Herpes simplex virus is a common viral CNS infection and presents suddenly with fever, headache, seizures,
and focal neurologic deficits. No Hx of insect bite.

-Measles encephalitis typically occurs following the initial illness with fever, cough, coryza, conjunctivitis, and
diffuse maculopapular rash. 

Schistosomiasis (6244)

Urinary symptoms, terminal hematuria, and peripheral eosinophilia, should raise suspicion for urinary
schistosomiasis.

Schistosomiasis is a parasitic blood fluke infection that affects more than 200 million people worldwide.  The
greatest prevalence is in sub-Saharan Africa.  Schistosomes live in specific types of freshwater snails, and

93
humans in endemic areas are often infected as children when bathing in contaminated freshwater.  Although
five different species of schistosomes cause human infections, urinary schistosomiasis is typically due to
Schistosoma haematobium.

Patients with urinary schistosomiasis are usually asymptomatic initially, but chronic infections are marked by
bladder inflammation and scarring with resultant dysuria, urinary frequency, and terminal hematuria. 
DX: Peripheral eosinophilia is seen in 30%-60% of cases, and anemia is common due to chronic blood loss. 
Diagnosis requires the identification of eggs using urine sediment microscopy.
RX: Treatment with praziquantel is usually curative.

Chronic schistosomiasis can be associated with bladder cancer, which usually presents with intermittent, gross,
painless hematuria that is present throughout micturition.  This patient with terminal hematuria, peripheral
eosinophilia, and dysuria first requires an evaluation for schistosomiasis using urine sediment microscopy.  If
this test is negative or his symptoms continue after treatment, an evaluation for bladder cancer using
cystoscopy with biopsy (the gold standard) would be prudent. 

Toxoplasmosis (5331)

Hydrocephalus, intracranial calcifications, and hepatomegaly are signs of congenital toxoplasmosis, which is
acquired by transplacental transmission.  Maternal infection is acquired via inadvertent ingestion of
Toxoplasma gondii from cat feces, contaminated soil on fruits/vegetables, or undercooked meat; infection
rates are highest in South America, where more virulent strains exist.  Routine screening is not undertaken in
the United States.

Although toxoplasmosis infection is typically asymptomatic in an immunocompetent adult, it can cause severe
consequences in the fetus or newborn.  After maternal ingestion, tachyzoites transmitted through the placenta
invade developing fetal cells, most commonly in the muscle and brain.  Classic congenital manifestations
include eye abnormalities (eg, chorioretinitis), neurologic findings (eg, intracranial calcifications,
hydrocephalus), and hearing impairment. 

DX: When infection is suspected prenatally, diagnosis is made via maternal serology or amniocentesis.  When
the infection is suspected in a newborn, diagnosis is with neonatal serology. 
RX: Treatment is with antiparasitic therapy (ie, pyrimethamine and sulfadiazine) and supplemental folate for a
year.
D/D

94
-Pertussis vaccination in pregnancy is recommended to promote passive immunity against this respiratory
illness. 

-Rubella infection in pregnancy is more commonly symptomatic (eg, fever, maculopapular rash) than
toxoplasmosis infection.  Congenital rubella typically manifests as congenital heart defects, eye abnormalities,
and hearing impairment.  Intracranial calcifications are not seen.

Skin and Tissue Infections (5598)

Erysipelas (5598)
Erysipelas, a specific form of cellulitis.  Currently, the lower extremities are the most common sites of
involvement, but the face is involved in 5 to 20% of patients.  The lesion is usually characteristic: it is red,
painful, edematous and elevated.  A sharp demarcation from the uninvolved skin is usually present, and a
butterfly pattern involving the cheeks and the bridge of the nose may be seen.  Other characteristic features of
erysipelas are an abrupt onset and the presence of systemic symptoms (fever, chills, malaise, etc).  Group-A
streptococcus is the typical etiologic agent of erysipelas, although sometimes, other beta-hemolytic
streptococci are involved.
RX: Treatment is a penicillin (eg, ampicillin, amoxicillin).

- Staphylococcus aureus and gram-negative organisms are very rare causes of the disease.

D/D

-Erysipelas differs from cellulitis in its rapid onset and well-demarcated and raised appearance. 

95
-Abscess presents with a tender, fluctuant nodule due to a collection of pus beneath or within the dermis. 
Purulent drainage and fever may be present.  This patient's rash is not fluctuant, and no purulent drainage is
present.

-Erythema migrans is a manifestation of Lyme disease and may present as a rash with fever and localized
lymphadenopathy.  However, the rash is typically not tender and presents as a bull's eye lesion with central
clearing.

-Impetigo is also a superficial bacterial infection but typically presents with pustules and honey-colored
crusting.  Systemic symptoms, such as fever and chills in this patient, are uncommon.

-Necrotizing fasciitis is also a rapidly spreading infection but classically presents with pain out of proportion
to examination findings, which is not present in this patient.  In addition, findings in necrotizing fasciitis can
include crepitus, bullae, and ecchymoses.

Cellulitis (13583)

96
Presentation includes tender and warm erythema with ill-defined, flat borders.  Infection typically develops
after a break in the skin, as in this patient with an insect bite.  The course is often indolent, with increasing size
and development of systemic symptoms (eg, fever) after several days.  Fluctuance and purulent drainage are
not present.

The most common cause of cellulitis is Streptococcus pyogenes, also known as group A Streptococcus. 

RX: Treatment is systemic antibiotic therapy (eg, cephalexin) for ≥5 days.  Well-appearing, afebrile patients
can be treated as outpatients.

D/D

-Staphylococcus aureus is the most commonly isolated pathogen in abscesses and is less commonly implicated
in nonpurulent cellulitis.  Although this patient is at risk for methicillin-resistant S aureus due to contact sports,
S pyogenes is still the most common cause of isolated cellulitis, and there is no fluctuance or purulent drainage
suggestive of an abscess.

-Trichophyton species can cause tinea corporis, a dermatophyte infection common in athletes who participate
in contact sports.  The rash is pruritic with central clearing and raised borders, and is not tender or warm.

Toxic Shock Syndrome (5865)

Most patients with TSS have a prodrome of fever, chills, and myalgias that evolves into a multisystemic
syndrome (eg, hypotension and shock with dermatologic, gastrointestinal, muscular, renal, and/or neurologic
findings); some patients can become abruptly ill within hours.  Almost all menstrual and most non-menstrual
cases of TSS are due to Staphylococcus aureus and are usually associated with a staphylococcal exotoxin
called TSS toxin-1.  The pathophysiology involves widespread activation of T cells by exotoxins acting as
superantigens (ie, can activate T cells directly without needing to be processed by antigen recognition cells). 
Massive cytokine production and release occur, leading to the observed clinical manifestations.

RX: They may require extensive fluid replacement, which can reach 20 L per day.  Physicians generally
administer clindamycin (which theoretically prevents toxin synthesis) with or without antistaphylococcal
antimicrobial therapy (vancomycin, or oxacillin or nafcillin if susceptible) to eradicate the organism and
prevent TSS recurrence. 

97
Necrotizing Fasciitis (5879)

Necrotizing fasciitis is a fulminant infection of the subcutaneous tissue that spreads rapidly along the fascial
planes and leads to extensive tissue necrosis and shock.

Two types of clinically distinct necrotizing fasciitis have been described.  The most common form (type II)
usually occurs in individuals with no concurrent medical illness.  Many patients report a history of laceration,
blunt trauma, or a surgical procedure as a predisposing factor.  It is typically caused by group A
Streptococcus (Streptococcus pyogenes). 

In contrast, type I is usually seen in patients with underlying diabetes and peripheral vascular disease.  It is
generally a polymicrobial infection; some commonly isolated organisms include Staphylococcus aureus,
Bacteroides fragilis, Escherichia coli, group A Streptococcus, and Prevotella species.  Crepitus is more
common if anaerobic organisms, such as Clostridium perfringens or B fragilis, are involved.

RX: Clinical suspicion of necrotizing fasciitis warrants urgent, aggressive surgical exploration and
debridement of the involved tissue to prevent morbidity and mortality.  These measures will also help to
ascertain the diagnosis by providing tissue samples for diagnostic purposes.  In addition to surgical
exploration, appropriate antibiotic therapy and hemodynamic support should be provided.

When the etiology of the necrotizing fasciitis is unknown, broad-spectrum therapy should be started.  This
includes:

1. Piperacillin/tazobactam or a carbapenem (eg, impinem, meropenem) will cover Group A


Streptococcus and anaerobes
2. Vancomycin will cover Staphylococcus aureus, including methicillin-resistant isolates
3. Clindamycin is added to inhibit toxin formation by streptococci/staphylococci

Once culture information is available, antibiotic therapy should be narrowed based on identified pathogens.

D/D

98
-C perfringens is capable of causing necrotizing fasciitis, either in isolation or in combination with other
bacteria.  Affected areas generally have crepitus on examination due to gas production by Clostridia. 

-Pseudomonas can cause necrotizing fasciitis in immunocompromised patients.  It would not be expected to
cause necrotizing fasciitis in an otherwise healthy young adult.

-S aureus can be seen as part of the polymicrobial etiology of type I necrotizing fasciitis in patients with poor
peripheral circulation, such as those with diabetes.  It is not usually associated with the development of
necrotizing fasciitis alone.

-Staphylococcus epidermidis is associated with bloodstream infections and is the most common cause of
central line-associated bloodstream infections.  It is not associated with necrotizing fasciitis.

Hand Mouth and Foot Disease (9988)

Herpangina is an oropharyngeal infection caused primarily by Coxsackie group A virus.  It is usually seen
during the summer in children age 3-10, who present with fever, drooling, sore throat, decreased appetite,
headache, and painful posterior pharyngeal vesicles.  The lesions can evolve rapidly from erythematous
macules to vesicles on the soft palate and tonsillar pillars, creating a painful pharyngitis.

DX: Diagnosis is based clinically on the enanthem. 

99
RX: Therapy is directed at symptomatic relief with saline gargles, analgesics, and antipyretics.  Cold, non-
acidic fluids or popsicles can be soothing and provide hydration.  Antiviral therapy is not indicated in this self-
limited illness, which typically resolves within a week.  Hand-washing by children and their close contacts can
prevent spread during outbreaks.

D/D

-Lesions of aphthous stomatitis are often larger and located on the anterior oral mucosa.  There are no fevers
or systemic symptoms with aphthous ulcers.

-Primary herpes simplex virus type-1 infection causes gingivostomatitis in children.  These patients tend to be
febrile and ill-appearing with vesicles on the anterior oral mucosa, gingivae, and perioral skin.

-The varicella zoster virus causes chickenpox, which generally presents with fever and a widespread vesicular
rash on the face, trunk, and extremities.  Lesions are usually pruritic and rarely affect the palms.

CF (6336)

100
Cystic fibrosis (CF) is an autosomal recessive disorder found most commonly in the Caucasian population. 
CF affects multiple organs (eg, pancreas, gastrointestinal tract), but pulmonary disease is the major cause of
morbidity and mortality.  CF lung disease is characterized by chronic bacterial colonization with periodic acute
exacerbations.  Staphylococcus aureus and Pseudomonas aeruginosa are the 2 principle pathogens.

Hyperinflation on chest radiograph is a sign of chronic obstruction from thickened secretions, and prominent
bronchovascular markings indicate concurrent inflammation and infection.

Empiric treatment of an acute exacerbation typically includes vancomycin for methicillin-resistant S aureus
coverage and 2-drug coverage for P aeruginosa.  Cefepime and ceftazidime are antipseudomonal
cephalosporins; amikacin and tobramycin are aminoglycosides that are also effective against P aeruginosa. 
Other effective agents include carbapenems, certain fluoroquinolones, aztreonam, and colistin.

Extended-spectrum penicillins with activity against P aeruginosa include piperacillin-tazobactam and


ticarcillin-clavulanic acid.

Other Drug Options:


-Ampicillin-sulbactam is a combination third-generation penicillin and beta-lactamase inhibitor commonly
used to treat intraabdominal infections.  This medication is not effective against P aeruginosa. 

-Ceftriaxone and azithromycin are the recommended antibiotics for community-acquired pneumonia requiring
hospitalization.  They will cover common causes of pneumonia (eg, Streptococcus pneumoniae, Mycoplasma
pneumoniae) but are not active against P aeruginosa.

- Moxifloxacin, a fluoroquinolone used as monotherapy for community-acquired pneumonia, has limited


antipseudomonal activity.  Metronidazole is an antibiotic that treats anaerobic and parasitic infections but is
also not effective against Pseudomonas.

-Trimethoprim-sulfamethoxazole is a combination antibiotic used to treat Pneumocystis jirovecii pneumonia,


not Pseudomonas.  P jirovecii pneumonia is an opportunistic infection found in immunocompromised patients
(eg, HIV) but would not be expected in a patient with CF.

101
102

You might also like